Practice Questions

Réussis tes devoirs et examens dès maintenant avec Quizwiz!

What are the prohibited transactions a disqualified person may not engage in?

1. self dealing fiduciary 2. an owner selling, exchanging, leasing, buying, lending, borrowing between disqualified persons 3. receipt of payment from a fiduciary when dealing with a party servicing the account (e.g. attorney, accountant) 4. transfer of income/assets to or for the benefit of a disqualified person 15% penalty/yr. penalty until corrected, but 100% if not corrected within a reasonable timeKent Reeder, age 52, works as the administrator and curator at the Museum of Antique Manuscripts, a not-for-profit organization in Metropolitan Center. He has worked there 18 years and began contributing to the 403(b) plan 12 years ago but skipped contributing last year. He earns $85,000 a year. He has asked you to maximize his contribution. Which of the following is/are TRUE? He may contribute $19,500 plus $6,500 for age 50+ catch-up, plus $3,000 long service catch-up. He may not contribute to the long-service catch-up this year due to omitting a contribution last year. He may contribute $19,500 plus $6,500 age 50+ catch-up. He may not participate in both the long service catch-up and the age 50+ catchup the same year. He is not eligible for the long service catch-up. III and V only. II only. I, III and IV only. I and III only.

Which of the statements below correctly describes the potential price volatility of the following five bonds? Bond A: A-rated; 7% coupon; matures in 10 years. Bond B: A-rated; 6% coupon; matures in 12 years. Bond C: A-rated; 7% coupon; matures in 12 years. Bond D: A-rated; 5% coupon; matures in 15 years. Bond E: A-rated; 6% coupon; matures in 15 years. Bond A has greater potential for price fluctuations than Bond C. Bond D has greater potential for price fluctuations than Bond E. Bond C has greater potential for price fluctuations than Bond B. Bond D has greater potential for price fluctuations than Bond B. I and II only. II and IV only. III and IV only. I, II, and IV only.

Choice "I" (Bonds A and C) have the same coupon, but Bond A has a shorter maturity therefore less volatility. Choice "II" (Bonds D and E) both mature in 15 years, but Bond D's lower coupon makes it more volatile. Choice "III" (Bonds C and B) both have the same maturity, but Bond B's lower coupon means it is more volatile. Choice "IV" (Bonds B and D) is a true statement due to both coupon and maturity. ! Lower coupon = more volatile ! Longer dated maturity = more volatile

A non-qualified deferred compensation plan providing the key employee with a vested beneficial interest in an account is known as: A Supplemental Executive Retirement Plan (SERP). A funded deferred compensation plan. An excess benefit plan. A Rabbi trust.

If the employee has a non-forfeitable beneficial interest in a deferred compensation account, the IRS considers the plan "funded" and subject to current income tax due because the employee has constructive receipt of the assets.

Your client has purchased stock with a margin position that required 50% initial margin and a 35% maintenance margin. The stock was originally valued at $23 per share when the transaction was undertaken and your client bought 1,000 shares. What stock price will trigger a margin call? $13.04 $17.69 $26.45 $32.86

The correct answer is B. Price = Loan ÷ (1 - MM) = (23 × . 50) ÷ (1 - .35) = 17.69

The Tax Reform Act of 1986 was roughly revenue neutral because: It was supported by both Republicans and Democrats. It was not intended to raise or lower taxes. It divided the tax burden evenly between individuals and businesses. It made the tax rates equal across all tax brackets. II only. I and III only. II and III only. I, II, and IV only.

The correct answer is A. A piece of tax legislation is considered revenue neutral when it is expected to neither raise nor lower the total amount of taxes to be collected.

During the peak of the economic cycle, which of the following should one undertake? Sell debt instruments Begin allocations to cash positions Buy debt instruments Sell gold and real assets I and II only. II and III only. III and IV only. I and IV only.

The correct answer is A. At the peak of the cycle, as the economy has reached full steam, it is an excellent time to sell not buy fixed (and generally lower return) instruments. It is also an excellent time to begin appropriations to cash in preparation for opportunities that may arise. Since inflation is still on (and rising after the peak), it often proves to be a good time to acquire metals rather than sell them.

Which of the following characteristics apply to paired plans (also known as "tandem plans")? Generally combines a money purchase pension and a profit-sharing plan. Actuarial assumptions required. Total contributions to the paired plans limited to 15% of payroll by IRC Section 404. Employer bears investment risk. I only. II and IV only. I, III and IV only. IV only.

The correct answer is A. Defined contribution plans do not require actuarial assumptions. Total contributions to both plans is limited to lesser of 100% or $57,000 (2020) by IRC Section 415. The profit-sharing plan is limited by IRC Section 404 to 25% of covered payroll. Employees bear the investment risk in DC plans.

Direct recognition programs used with life insurance policies are best described in the following statement: Any amount of cash that is removed from the policy is reflected in a decrease in the amount of dividends and interest paid on that policy. Mutual companies that are owned by their policy holders directly pay profits to the policy owners. Very large policies indicate a recognized tendency of the company to write primarily term insurance. If the agent has received many awards from his company, he would be a good one to select.

The correct answer is A. Direct recognition programs are best described as follows: Any amount of cash that is removed from the policy is reflected in a decrease in the amount of dividends and interest paid on that policy.

Generally, which of the following are noncontributory plans? 401(k) and money purchase pension plans . 401(k) and thrift plans. Thrift plans and ESOPs. Money purchase pension plans and profit sharing plans. IV only. I and II only. III and IV only. I, II, III and IV.

The correct answer is A. Employers generally contribute to Money Purchase Pension Plans, ESOPs, and Profit Sharing Plans. These are considered non-contributory. Employees contribute (thus contributory plans) to 401(k)s and Thrift Plans.

Which of the following is not an example of a form of indirect federal regulation over the insurance industry? FDIC: Federal Deposit Insurance Corporation. IRC: Internal Revenue Codes. SEC: Securities Exchange Commission. ERISA: Employees Retirement Income Security Act.

The correct answer is A. FDIC is coverage for bank deposits. The rest are indirect federal government involvement.

George, age 35, works for XZY Brothers, Inc., which is installing a new SIMPLE IRA plan in the current year with the maximum match for this year. George makes $30,000 per year and is eligible to participate in the plan. Which of the following is true? George can have a maximum of $14,400 placed into his account this year. George may only have a maximum of $4,500 placed into his account this year. George may have a maximum of $13,500 placed in his account this year. George may have a maximum of $7,500 placed in his account this year.

The correct answer is A. George can put in 100% of salary up to $13,500 (2020). XYZ will match dollar for dollar up to 3% of salary ($30,000 x .03 = $900). So a total of $14,400 will be placed into the account. All other statements are false.

Kent Reeder, age 52, works as the administrator and curator at the Museum of Antique Manuscripts, a not-for-profit organization in Metropolitan Center. He has worked there 18 years and began contributing to the 403(b) plan 12 years ago but skipped contributing last year. He earns $85,000 a year. He has asked you to maximize his contribution. Which of the following is/are TRUE? He may contribute $19,500 plus $6,500 for age 50+ catch-up, plus $3,000 long service catch-up. He may not contribute to the long-service catch-up this year due to omitting a contribution last year. He may contribute $19,500 plus $6,500 age 50+ catch-up. He may not participate in both the long service catch-up and the age 50+ catchup the same year. He is not eligible for the long service catch-up. III and V only. II only. I, III and IV only. I and III only.

The correct answer is A. He is not eligible for the long service catch-up because the museum is not a Health, Education, Religious (HER) organization. The maximum contribution limits for 2020 are $19,500 plus the age 50+ catch-up of $6,500.

Matt is a participant in a profit sharing plan which is integrated with Social Security. The base benefit percentage is 6%. Which of the following statements is/are true? The maximum permitted disparity is 100% of the base benefit level or 5.7%, whichever is lower. The excess benefit percentage can range between 0% and 11.7%. Elective deferrals may be increased in excess of the base income amount. The plan is considered discriminatory because it gives greater contributions to the HCEs. I and II only. I, II and IV only. II only. I, II, III and IV.

The correct answer is A. His base rate is 6% and the social security maximum disparity is 5.7% for 11.7% as the top of his range. Statement "III" is incorrect because integration does not affect voluntary deferrals by employees. Statement "IV" is incorrect because, done properly, integration is NOT considered discriminatory.

What is one reason a company may call bonds that were previously issued? The bonds are currently selling at a premium. The bonds are currently selling at a discount. The company expects interest rates to decrease. The bonds are selling at par.

The correct answer is A. If the bonds are selling at a premium, then interest rates have decreased since the bonds were issued. The company would be motivated to retire the higher yield bonds and issue new bonds at lower market interest rates. A discount bond would indicate that interest rates of increased and the bond is paying a lower rate than current market interest rates.

When a property claim has been submitted, the adjuster is called in to do which of the following: Assist the insured in the preparing the proof-of-loss statement. Determine whether there was a loss covered by the policy. Classify the loss as standard, substandard, or ineligible. Choose the arbitrator who will determine the amount of loss. I and II only. I and III only. I and IV only. II and III only.

The correct answer is A. Options "I" and "II" are roles of an adjuster. Option "III" is incorrect because the classification is the underwriter's job. Option "IV" is incorrect because the adjuster determines the amount of loss. An arbiter would enter into the situation only if the two sides cannot agree.

Jerome is covered under his employer's money purchase pension plan. Several things happened in the current year. Which of the following would increase the company contributions for the current year? The company gave all key employees a 5% raise and all non-key employees a 3% raise. One of the key employees retired. The company had two employees terminate who forfeited a total of $10,000. The forfeitures were allocated to the remaining participants. The equity market declined and all account balances declined by at least 2%.

The correct answer is A. If the company gave everyone a raise then that would increase the company's contributions. If a key employee retired and two employees left that would decrease the company's contributions. Since the forfeiture allocations were allocated to the participants they would have no effect of the company's contributions. In a money-purchase pension plan the investment risk is on the employees and thus an increase or decrease in the investments has no impact on contributions.

Bond A has a 6% annual coupon and is due in 2 years. Its value in today's market is $900. Bond B has a 10% annual coupon and is due in 4 years. It is priced to yield 12%. Bond C is a zero-coupon bond priced to yield 11% in 8 years. Assuming the duration of Bond A is 1.94 years, which of the following statements about the effect of a 1% decline in interest rates is true? Bond C, having a longer duration that Bond A, would have a larger percent increase in price than Bond A. The percent change in price of a bond is independent of the duration of a bond. It is NOT possible to determine the percent change in price of Bond A versus Bond C because the duration of Bond C is NOT given. Bond A would have a greater percent change in price than Bond C because it has a shorter duration.

The correct answer is A. In this instance, there's no need to do any calculations. They're already (inherently) done. Because it gives the duration of Bond A (directly) and Bond C (indirectly as zero coupon bonds always have a duration equal to their maturity), the longer duration bond (Bond C) will have the greatest sensitivity to interest rate changes (because of the longer duration). Bond B is ignored simply because it's not found in the answers as a choice. Thus, answer A is correct. Answer B is not correct because the percent price change is dependent on duration. Answer C is not correct because duration is given (indirectly by giving the maturity and the fact it's a zero). Answer D is incorrect simply because it's not true of duration. If you choose to calculate: Multiply the change of rate by the duration to arrive at the percent change in price (see formula sheet for percent change formula). For each percent of change, multiply by the duration to arrive at the total percent change. Then calculate that percentage into a price change. **This may exceed the 1.5 minutes per question suggested on the exam.

Jacinth is an executive at Papers Unlimited. As part of her compensation she has a restricted stock plan that allows her to receive 2,000 shares of stock after she completes of 5 years of service. At the time of grant the stock was trading at $4 per share. She did not make the 83b election. She met the vesting requirement 8 months ago when the stock was trading at $28. She has decided to sell her stock. All of the following are true, except: If she sells the stock today for $3 per share she would have an ordinary loss of $50,000. If she sells the stock today for $15 per share she will recognize a capital loss of $26,000. If she sells the stock today for $28 per share then she will not recognize any gain or loss. If she sells the stock today for $38 per share then she will recognize $20,000 in short term capital gains.

The correct answer is A. In this question you are looking for the false statement. When she met the vesting period she would have recognized W-2 income of $56,000 (2,000 × $28). A is the false statement: If she sold the stock at $3 then she would recognize a capital loss of $50,000 ($56,000 basis - $6,000 sale price). Loss from an investment must be netted against gain. If the loss exceeds the capital gain, $3,000 can be taken against ordinary income, not the full amount. If she sold the stock at $15 then she would recognize a capital loss of $26,000 ($56,000 basis - $30,000 sale price). If she sold the stock at $28 then she would not recognize any gain or loss ($56,000 sale price - $56,000 basis). If she sold the stock at $38 then she would recognize a short term gain of $20,000 ($76,000 sale price - $56,000 basis).

Jack is a greeter at Doogle Mart and works full time. Unfortunately, he does not own any of the stock as the business has increased significantly over the years. He turned 70 ½ on November 19th (2019) of year 2. The table below reflects the balance in each of his retirement accounts at the end of each specified year. He no longer works for Walton Company. Assume the life expectancy factor for ages 70, 71, 72, and 73 are 27.4, 26.5, 25.6 and 24.7 respectively. What is the minimum distribution for Jack for year 2, exclusively? Dec 31 Year 1Dec 31 Year 2Dec 31 Year 3Dec 31 Year 4IRA # 1 Traditional $175,000 $190,000 $210,000 $220,000IRA # 2 Traditional $100,000 $100,000 $120,000 $130,000IRA # 3 Roth $125,000 $130,000 $140,000 $150,000401(k) Walton Co. Roth Account $100,000 $120,000 $130,000 $140,000Profit Sharing Plan Walton $100,000 $110,000 $110,000 $120,000401(k) Doogle Mart Co. $150,000 $180,000 $190,000 $210,000Profit Sharing Plan Doogle Mart Co. $250,000 $280,000 $290,000 $300,000TOTAL $1,000,000 $1,110,000 $1,190,000 $1,270,000 $17,336 $22,642 $32,075 $37,736

The correct answer is A. Minimum distributions must be taken by Jack for all traditional IRAs and all qualified plans other than Doogle Mart, since he is still employed there he can defer . His attained age at the end of year 2 is still 70 because he will not turn 71 until May of year 3, which equates to a factor of 27.4. The total balances from the end of year 1 total $475,000, which is $1 million less the balance for the Roth IRA and the $400,000 in the Doogle Mart. $475,000/27.4=17,336. The Roth 401(k) is subject to RMDs. The employee contributions are only characterized as Roth if the employee elects. The employer matching is always treated as traditional 401(k) contributions.

Which of the following statements about preferred stock are true? Its market fluctuations are greater than the long-term bond market fluctuations. It is more risky than debt. Its dividends are recomputed quarterly. It has no interest rate risk because it is a stock and not a bond. I and II only. I and IV only. II and III only. II and IV only.

The correct answer is A. Option "III" - Dividends are set at issue by the Board as a percent of par value and do not change. Option "IV" - Changes in interest rates directly impact preferred stock, and there is no relief on preferred stock because most cannot be held to maturity (as most are issued without a maturity date). Preferred stock is more risky than bonds because bonds are a legal obligation and have a higher priority in bankruptcy proceedings.

Of the following indexes, which is the only one that uses the geometric average to compute its daily value? NASDAQ Index. Wilshire 5000 Index. Value Line Average. Dow Jones Industrial Average.

The correct answer is C. The NASDAQ, the NYSE Composite, and the Wilshire all use value weighted average, while the Dow Jones Industrial is a simple price weighted average. Only Value Line uses the geometric average.

Which of the following accurately describe the results of "golden parachute" payments made to a "disqualified" person? They are includible in W-2 income. They are subject to a 10% excise tax. They qualify for 10-year forward averaging if paid out as a lump sum. They are not subject to payroll taxes. I only. II only. II and III only. I and IV only.

The correct answer is A. Payments under a "golden parachute" are considered ordinary income. Additionally, any amounts under the Social Security cap will be subject to the OASDI tax. All amounts will be subject to Medicare tax. "Golden parachute" payments are also subject to an additional 20% excise tax. Because these are non-qualified plans, no lump sum treatment or IRA rollover options apply.

An investor may use options on debt instruments to protect against: Interest rate risk. Reinvestment rate risk. Default risk. Call risk.

The correct answer is A. Put options that lock in the price at which the security may be sold may be used to protect an investor from a drop in bond prices caused by rising interest rates.

Ernest converted his Traditional IRA to a Roth IRA on Dec 15, 2014. He was 35 years of age at the time and had never made a contribution to a Roth IRA. The conversion was in the amount of $60,000 ($10,000 of contributions and $50,000 of earnings). Over the years he has also made $15,000 in contributions. On May 15, 2018 he withdrew the entire account balance of $100,000 to pay for a 1 year trip around the world. Which of the following statements is true? $25,000 of the distribution will be subject to income tax and $85,000 of the distribution will be subject to the 10% early withdrawal penalty. $25,000 of the distribution will be subject to income tax and the 10% early withdrawal penalty. Some of the distribution will be taxable but the entire distribution will be subject to the 10% early withdrawal penalty. None of the distribution will be taxable nor will it be subject to the 10% early withdrawal penalty.

The correct answer is A. Roth distributions are tax free if they are made after 5 years and because of 1)Death, 2)Disability, 3) 59.5 years of age, and 4)First time home purchase. He does not meet the five year holding period or one of the exceptions. His distribution does not received tax free treatment. The treatment for a non-qualifying distribution allows the distributions to be made from basis first, then conversions, then earnings. His basis will be tax free. The conversion is also tax free since we paid tax at the time of the conversion on those earnings. The remaining earnings since establishment of the Roth are $25,000 (100,000 - $15,000 in basis - $60,000 in conversions) and will be taxed. The 10% penalty does apply to this distribution since he does not qualify for any of the exceptions to the penalty. The contributions escapes penalty but the conversions and earnings of $85,000 are subject to the 10% early withdrawal penalty. Remember that in order for the conversions to escape the 10% early withdrawal penalty the distribution must occur after a 5 year holding period beginning Jan 1 in the year of conversion or meet one of the 10% early withdrawal exceptions.

A supplemental deferred compensation plan providing retirement benefits above the company's qualified plan AND without regard to Section 415 limits is known as: A Supplemental Executive Retirement Plan (SERP). A funded deferred compensation plan. An excess benefit plan. A Rabbi trust.

The correct answer is A. SERP supplements the pension plan without regard to limits imposed upon salary levels (i.e., maximum salary of $285,000 in 2020) or the maximum funding levels of Section 415. Do not confuse with an excess benefit plan which extends the benefits of a company's qualified plan above the Section 415 limits but still adheres to maximum salary limitations.

Cody is considering establishing a 401(k) for his company. He runs a successful video recording and editing company that employs both younger and older employees. He was told that he should set up a safe harbor type plan, but has read on the Internet that there is the safe harbor 401(k) plan and a 401(k) plan with a qualified automatic contribution arrangement. Which of the following statements accurately describes the similarities or differences between these types of plans? The safe harbor 401(k) plan has more liberal (better for employees) vesting for employer matching contributions as compared to 401(k) plans with a qualified automatic contribution arrangement. Both plans provide the same match percentage and the same non-elective contribution percentage. Employees are required to participate in a 401(k) plan with a qualified automatic contribution arrangement. Both types of plans eliminate the need for qualified matching contributions, but may require corrective distributions.

The correct answer is A. Safe harbor plans require 100% vesting, while 401(k) plans with QACAs require two year 100% vesting. The matching contributions are different for the plans. Employees are not required to participate in either plan. Both plans eliminate the need for ADP testing, which means that they eliminate the need for qualified matching contributions and corrective distributions.

Bertha, who is 54 years old, spent most of her career in the corporate world and now provides consulting services and serves as a director for several public companies. Her total self-employment income is $500,000. She is not a participant in any other retirement plan today. She would like to shelter as much of her self-employment earnings as possible by contributing it to a retirement plan. Which plan would you recommend? Establish a 401(k) plan. Establish a target benefit plan. Establish a Deferred Comp program for herself. Establish a SEP.The correct answer is A. She will be able to defer $57,000 plus the catch up of $6,500 to the 401(k) plan, where she can only contribute $57,000 to the target benefit plan and the SEP. She cannot set up a deferred compensation plan and defer tax.

The correct answer is A. She will be able to defer $57,000 plus the catch up of $6,500 to the 401(k) plan, where she can only contribute $57,000 to the target benefit plan and the SEP. She cannot set up a deferred compensation plan and defer tax.

Shane's Rib Shack has a Target Benefit Plan. They have 10 employees with the following compensations: Employee Compensation $300,000 $100,000 $75,000 $50,000 $50,000 $50,000 $50,000 $25,000 $25,000 $20,000 Based on the actuarial table that was established at the inception of the plan they should fund the plan with $210,000. What is the maximum deductible contribution that can be made to the plan? $182,500 $186,250 $195,000 $210,000

The correct answer is A. Since the plan is a defined contribution plan the maximum deductible contribution is 25% of the total covered compensation. The max covered compensation of all employees is $730,000. Thus the maximum deductible limit is $182,500 ($730,000 × 25%). Remember to limit employee 1 to the $285,000 (2020) covered compensation limit. The actuarial table amount is irrelevant because this a defined contribution plan.

Sammy, age 67, purchased a qualified long-term care policy 14 years ago. The policy currently pays him a $125 daily benefit. His adjusted gross income is $60,000, and he has incurred unreimbursed medical expenses of $7,000 this year. In addition, he pays $3,000 per month to stay in a nursing home. Based on this information, the long-term care benefit would be: A. Completely nontaxable. B. Partially taxable, based on his age. C. Partially taxable, based on his adjusted gross income. D. Fully taxable.

The correct answer is A. Since the policy is qualified, the long-term care benefit received is nontaxable.

Robin purchased a mutual fund at NAV of $20.00 and sold it 8 months later at $21.00. During the time he owned the fund, he received a LTCG of $1.00/share and a qualified dividend distribution of $.75/share from the mutual fund. He has a marginal tax rate of 32%. The tax on LTCG is 15%. What is his after-tax holding period return? 10.84% 11.5% 11.8% 12.3%

The correct answer is A. Since this is a ST holding period, it's ordinary income at the marginal tax rate for the price increase. Since the dividend distribution is a qualified dividend, it receives capital gians tax treatment. HPR = (SP - PP +/- CF) × (1-TR) / PP [($21.00 - $20.00) × (1 - .32)] + [($1.00 + $.75) × (1 - .15)] / $20.00 Answer: 10.84% (remember to follow order of operations: Parenthesis, exponents, multiplication, division, addition, subtraction) Based on Subchapter M or pipeline theory, investment companies must payout at least 90% of their portfolio earnings. If a mutual fund sells a position they hold at a gain, it passes the gain, like-kind, to its investors.

Which of the following statements accurately reflects the overall limits and deductions for employer contributions to qualified plans? An employer's deduction for contributions to a money purchase pension plan and profit sharing plan is limited to the lesser of 25% of covered payroll or the maximum Section 415 limits permitted for individual account plans. An employer's deduction for contributions to a defined benefit pension plan and profit sharing plan cannot exceed the lesser of the amount necessary to satisfy the minimum funding standards or 25% of covered payroll. Profit sharing minimum funding standard is the lesser of 25% or the Section 415 limits permitted for individual account plans. I only. I and II only. II and III only. I, II and III.

The correct answer is A. Statement "II" is incorrect because there is no 25% of covered payroll limitation in a DB plan. Statement "III" is incorrect because there is no minimum funding standard for profit sharing plans.

Myron has a life insurance policy in his qualified plan at work. He has come to you for advice about retirement and other financial planning needs. Which of the following is not correct about the life insurance in a qualified plan? He will be subject to income only if the policy in his qualified plan is a cash value type policy. The policy will be included in his gross estate if he were to die while still working. Part of the proceeds could be taxable to his beneficiary if it is a cash value policy. When he distributes the policy from his plan at retirement, he can convert it to an annuity within 60 days to avoid taxation.

The correct answer is A. Statements b, c, and d are correct. Statement a is false because all life insurance in qualified plans is subject to income when purchased, regardless of the type.

A 56-year-old client becomes unemployed due to a disability. The client tells a CFP® professional that he hopes to go back to work eventually, but is not sure when that might be. Until then, he needs to generate replacement income. His only available asset is his traditional 401(k) plan. What is the best way for the client to replace his income (CFP® Certification Examination, released 8/2012)? Directly from the 401(k) plan From a rollover IRA, using Rule 72(t) From a rollover annuity, using substantially equal payments From a brokerage account, using net unrealized appreciation (NUA)

The correct answer is A. The 401(k) plan is penalty-free and will not require continued payments if the client goes back to work.

Which of the following capital needs analysis methods mitigates the risk of outliving retirement funds? Capital Preservation Model Present Value of an Annuity Due Model Purchasing Power Preservation Model Serial Accumulation Model

The correct answer is A. The Capital Preservation Model assumes at life expectancy, as estimated in the annuity method, the client has exactly the same account balance as he/she started with at retirement. So if life expectancy is exceeded there is still capital available.

Which of the following is not a requirement for the owner of corporate stock who sells to an ESOP to qualify for the nonrecognition of gain treatment? The ESOP must own at least 55% of the corporation's stock immediately after the sale. The owner must reinvest the proceeds from the sale into qualified replacement securities within 12 months after the sale. The ESOP may not sell the stock within three years of the transaction unless the corporation is sold. The owner must not receive any allocation of the stock through the ESOP.

The correct answer is A. The ESOP must own at least 30% of the corporation's stock immediately after the sale. All of the other statements are true.

A convertible bond has the following terms: Maturity value = $1,000 Time to maturity = 20 years Coupon rate = 8% Call penalty = One year's interest Exercise price = $10 per share. Given this information, you will inform your client as to how many shares of stock that the bond may be converted. 100 shares 80 shares 50 shares 20 shares

The correct answer is A. The conversion ratio is used here (PAR/Conversion Price) to determine how many shares are available upon conversion. $1,000 / $10 = 100 shares.

Which of the following statements concerning the OASDHI earnings test for the current year is correct? Some part-time work is allowed without the loss of retirement benefits for those under normal age retirement. The earnings test does not apply after the age of 62. Interest and dividends are included in the earnings test. The annual exempt amount for a person at normal age retirement is $48,600.

The correct answer is A. The earnings test does not apply at, or after, normal age retirement. The monthly exempt amount is $4,050 ($48,600) in 2020 for those months in the year of normal retirement age BEFORE you actually reach normal retirement age. The test uses only earned income. No passive or portfolio income is used in calculating the earnings.

Jane P. Lane is a clerical worker who has been with her employer for the last 20 years. Last year, she got married in the Swiss Alps, which was quite out of character for her. She participates in an employer-paid group term life plan and selected term insurance in the amount of $200,000, which is three times her salary. She has named her spouse as the beneficiary of the policy. What is the tax consequence of this policy? Her employer is permitted to deduct the premiums paid on the entire amount of coverage. Her employer is permitted to deduct the premiums paid on the first $50,000 of coverage. Jane is subject to tax on the entire benefit. Jane is subject to tax on the amount which exceeds three times her annual salary or $50,000, whichever is less.

The correct answer is A. The employer is permitted to deduct 100% of the premiums, but Jane is subject to taxation on the amount in excess of $50,000.

One of the five tests which must be met to qualify as a dependent is: Whether the dependent files a joint return that year. That the dependent owes taxes from a previous year. Will the dependent owe taxes this year. The unearned income reported by the dependent.

The correct answer is A. The five dependency tests are: 1) Gross Income Test, 2) Support Test, 3) Member of Household or Family Member Test, 4) Citizenship Test (U.S., Canada or Mexico), and 5) Joint Filing Test.

Walt Drizzly stock is currently trading at $45 and pays a dividend of $3.50. Analysts project a dividend growth rate of 5%. Your client, Toby Benjamin, requires a rate of 12% to meet his stated goal. Toby wants to know if he should purchase stock in Walt Drizzly. Yes, the stock is undervalued. No, the stock is overvalued. No, the required rate of return is higher than the projected growth rate. Yes, the required rate is higher than the expected rate.

The correct answer is A. The intrinsic value is: V = (D1 / r - g), therefore V = (3.50 × 1.05) / (.12 - .05), V = $52.50 compared to the selling price of $45. Therefore the stock is undervalued.

What is the main responsibility of the underwriting department of a life insurance company? To guard against adverse selection. To set a limit on the amount of insurance issued. To set adequate insurance rates. To avoid exposures that could result in loss.

The correct answer is A. The limit on the amount of insurance (Option "B") is determined by company policy. Adequate insurance rates (Option "C") is set by the state. In regard to Option "D", it is the nature of insurance to cover losses.

Which of the following statements accurately reflect the characteristics of a Section 457 plan? Benefits taken as periodic payments are treated as ordinary income for taxation. Lump-sum distributions are eligible for 5-year and/or 10-year averaging. Deferred amounts are subject to Social Security and Medicare taxes at the later of: performance of services or employee becomes vested in the benefits. Income tax withholding is not required until funds are actually received as opposed to constructively received. Cannot exceed the smaller of $19,500 or 100% includible compensation in 2020. I, III and V only. II, IV and V only. I, II, IV and V only. I, II, III, IV and V.

The correct answer is A. There are no special tax advantages provided for 457 plans distributed in a lump-sum. Income tax withholding is required once the benefits are constructively received, even if not actually received.

Which of the following is NOT a premium factor that would be considered part of the nominal rate of interest? Economic premium. Default premium. Liquidity premium. Risk free rate of interest.

The correct answer is A. There is no such thing as an economic premium. All of the other premiums added to the risk free rate equal the nominal (or stated) rate.

Sara and Bill have rental property that was rented this year to a family whose primary bread winner lost his job. As a result they had uncollected rent for 2 months before they began the eviction process and 1 additional month before the family was finally evicted. If the rent was $800 per month, how much of a deduction may Sara and Bill claim on their income tax return for the uncollected rent?

The correct answer is A. They cannot deduct from ordinary income an amount that was never included in taxable income. Therefore, Sara and Bill will not receive any deduction for the uncollected rents.

You are faced with several fixed income investment options. Which of these bonds has the greatest reinvestment rate risk? A U.S. Treasury bond with an 11.625% coupon, due in five years with a price of $1,225.39 and a yield to maturity of 6.3%. A U. S. Treasury strip bond (zero-coupon) due in five years with a price of $735.12 and a yield to maturity of 6.25%. A corporate B-rated bond with a 9.75% coupon, due in five years with a price of $1,038.18 and a yield to maturity of 8.79%. A corporate zero coupon bond due in 5 years with a price of $750 and a yield to maturity of 5.9%.

The correct answer is A. This is due to the high coupon and lack of similar rates currently.

Jenny bought 250 shares of XYZ stock at $30 per share, with an initial margin of 60%. She paid 8% margin interest annually. One year later she sold all of the stock for $10,500. Jenny is in the 35% marginal tax bracket and itemizes her deductions. What is Jenny's holding period yield? 40% 61.33% 51.47% 34%

The correct answer is A. This question asks for holding period yield, not holding period return. To calculate the holding period yield, you simply subtract the purchase price from the selling price and divide by the purchase price (($10,500 / 250) - $30 )/ $30. Choice B is not correct because that is the calculation for holding period return. Choice C is not correct because that is the calculation for the after-tax holding period return. Choice D is not correct because that is the calculation for the after-tax holding period return without the margins.

Your client Bebe Rebozo is contemplating the exchange of two parcels of investment land for two similar parcels in two separate transactions. Given the following details of the proposed transaction, compute the amount of recognized gain and loss (if any) on both parcels if your client completes the exchanges: Parcel A: Ten acres of land acquired 15 years ago with a current basis of $50,000. In exchange your client will receive eight acres of land (FMV = $80,000) and $20,000 in cash. Parcel B: Twenty acres of land acquired two years ago with a current basis of $100,000. In exchange, your client will receive twelve acres of land (FMV = $75,000) and $10,000 in cash. Parcel A Recognized Gain = $20,000; Parcel B Recognized Loss = $0 Parcel A Recognized Gain = $20,000; Parcel B Recognized Loss = $10,000 Parcel A Recognized Gain = $50,000; Parcel B Recognized Loss = $10,000 Parcel A Recognized Gain = $20,000; Parcel B Recognized Loss = $15,000

The correct answer is A. This question pertains to like kind exchanges where boot is involved. The rule is that any realized gain will be recognized to the extent of the lesser of realized gain or boot received. In this case, there was a realized gain of $50,000 ($50,000 basis for $100,000 market value). The boot of $20,000 is recognized as gain since it is the lesser of boot or realized gain. Parcel B will have no gain in that there is no realized gain between the basis of the property given up and the fair market value of the property received. There is a realization but it is not recognized. Losses in like kind exchanges are not recognized.

Jan Grimlaw is interested in purchasing a municipal bond that pays $35 interest on a semi-annual basis. The bond is selling at par of $1,000 and Jan is in the 28% marginal tax bracket. She would like to know what pre-tax yield she would need to receive to have a comparable corporate bond investment. 9.72% 8.96% 7.35% 6.28%

The correct answer is A. Use the tax equivalent yield formula to calculate the correct answer. TEY = (Tax Exempt Rate) ÷ (1 minus investor's marginal tax rate) = .07 ÷ (1 - .28) = .0972 or 9.72% Calculate the municipal bond's coupon rate: $35 × 2 = $70 70 ÷ 1,000 = .07

Maria, age 28, has just expressed an interest in retiring at age 55 and having an income of the equivalent of $40,000 per year in retirement income in today's dollars. She assumes that she can make 8% interest after tax and expects inflation to average about 4% per year. Her life expectancy is 85 years old and she wants to know how much she should be saving each year in her savings plan to reach her goal between now and her retirement. $7,625 $24,159 $8,068 $15,311

The correct answer is B HP10BII and HP 10BII+: Step #1 - NPV at time period zero: 0 CFj, 0 CFj, 26 shift Nj, 40,000 CFj, 30 shift Nj, 1.08/1.04 = -1 = × 100 = i/yr, Shift NPV gives $264,184.34; Step #2 - Annual savings required: N = 27, I = 8, PV = $264,184.34, PMT = ?, FV = 0, Answer: $24,159.

Prince, age 60, is the sole member of Symbols, LLC. Symbols sponsors a 401(k) / profit sharing plan. Prince's self-employment income (after expenses) was $123,000 and his self-employment taxes were $17,400 for the year. What is the maximum that could be contributed by the employer or Prince for the benefit of Prince for 2020? $42,360 $48,860 $49,600 $55,850

The correct answer is B. $123,000Net Income(17,400÷2 = $8,700)Less 1/2 SE Tax$114,300Net SE Income× 0.20(0.25 ÷ 1.25)$22,860Employer contribution$19,500Plus 401(k) Deferral (employee elective before deferral)$6,500Plus > age 50 catch up$48,860Total

Rick and Amber (husband and wife), residents of a non-community property state, owned unimproved land that they have titled in joint tenancy with rights of survivorship. Rick purchased the land with his own funds for $100,000 five years ago, and he died in the current year when the land was worth $400,000. What is the amount associated with the land that will be included in Rick's gross estate? $100,000. $200,000. $300,000. $400,000

The correct answer is B. 50% of the fair market value must be included in Rick's estate because of the deemed contribution rule because his joint tenant Amber is his spouse. If he titled JTWROS with anyone but a spouse we would use the "actual contribution rule" in which case he would have $400,000 included in his gross estate. Note that is he had titled the property tenants in common, he would have had $400,000 inclusion. Actual contribution rule applies to TC and JTWROS if not spouse.

Which of the following statements is correct regarding a Section 457(b) plan? A. It is a type of qualified plan. B. It features a catch-up adjustment for individuals age 50 or older. C. Participants in the plan are considered active participants for purposes of determining deductibility of contributions to a traditional IRA. D. Employer contributions to the plan are deductible by the employer, if made by the extended due date of the employer's tax return.

The correct answer is B. A is incorrect. A Section 457(b) plan is a type of nonqualified plan. C is incorrect. Since the Section 457(b) plan is a type of nonqualified plan, individuals participating in the plan are not considered active participants. D is incorrect. Section 457(b) plans are sponsored by tax-exempt organizations. Tax-exempt organizations do not have income tax deductions.

The Health Insurance Portability and Accountability Act of 1996 (HIPAA) impacts an employee and employer in which of the following ways: An employee without creditable coverage can generally only be excluded by the group health insurance plan (if offered) for up to twelve months. The waiting period is reduced by the amount of "creditable coverage" at a previous employer. If the employee does not enroll in the group health insurance plan at the first opportunity, an 18-month exclusion period may apply. I and II only. I, II and III only. II and III only. II only.

The correct answer is B. All three statements are true. If you have a pre-existing condition that can be excluded from your plan coverage, then there is a limit to the pre-existing condition exclusion period that can be applied. HIPAA limits the pre-existing condition exclusion period for most people to 12 months (18 months if you enroll late), although some plans may have a shorter time period or none at all. In addition, some people with a history of prior health coverage will be able to reduce the exclusion period even further using "creditable coverage." People with a history of prior health coverage will be able to reduce the exclusion period even further using "creditable coverage."

How do cash balance plans differ from traditional defined benefit pension plans? Traditional defined benefit plans are required to offer payment of an employee's benefit in the form of a series of payments for life while cash balance plans are not. Traditional defined benefit plans define an employee's benefit as a series of monthly payments for life to begin at retirement, but the cash balance plan defines the benefit in terms of a stated account balance. In Cash Benefit Plans, these accounts are often referred to as hypothetical accounts because they do not reflect actual contributions to an account or actual gains and losses allocable to the account, whereas in a Defined Benefit Pension Plan they do. Pension Plans are available to retirees in a lump sum payment, whereas Cash Balance Plans are not.

The correct answer is B. Answer "A" is incorrect because Cash Balance Plans are required to offer payment of an employee's benefit in the form of a series of payments for life. Answer "C" is incorrect, because neither plan shows actual gains or losses allocable to the account. Answer "D" is incorrect and stated exactly opposite of how it is in fact.

Bob Jones is a senior executive at Sys, a global outsourcing firm based in New York. The company has recently rolled out a new compensation program that includes non-qualified stock options. Bob has approached you, as his planner, with several questions about the tax impact involved in his nonqualified (non-statutory) stock options. His circumstance is as follows: February 1st: he was granted an option when the company stock price was $30. His option exercise price was $30, but option value was not readily ascertainable. August 1st: he exercised the option when the stock price was $50. September 1st: with the price at $50 per share and seeming to have peaked, Bob is wondering if he should sell the stock. Which of the following applies? Bob will have ordinary income of $50 if he sells at a stock price of $50. Bob will have ordinary income at exercise, based on the spread between the stock price and the exercise price of the option. Bob will have long-term capital gain income of $20 if he sells the stock on September 1st at a price of $50. Bob will have ordinary income of $30 when the option is awarded.

The correct answer is B. Bob will not be taxed for gain upon the award of the option because of the lack of readily ascertainable value. He will be taxed at exercise on the spread between the stock price of $50 and exercise price of $30, and finally, upon sale of the stock for any additional amounts of gain.

What is the portfolio deviation of a portfolio invested 60% in stock "A" with a 15% return and a deviation of 17.5%, and the balance in stock "B" with an 18% return and a 16.75% deviation. There is a .29 correlation between the two securities. 16.2% 14.0% 13.05% 4.69%

The correct answer is B. COV = .175 × .1675 × .29 = .0085 s p = √(.60)2(.175)2 + (.40)2(.1675)2 + 2(.60)(.40)(.0085) s p = √.011 + .0045 + .0041 s p = √.0196 s p = .1399

You are about to recommend international mutual funds to your clients. Which of the following will you tell your clients are advantages of investing internationally? International markets are less efficient than U.S. markets. International mutual funds do not have the exchange rate risks of individual foreign stocks. Due to lower correlations with U.S. stocks, foreign stocks can lower total portfolio risk. Investors in foreign securities avoid U.S. tax on realized capital gains. I and II only. I and III only. II and IV only. I, III and IV only.

The correct answer is B. Choice "I" - Less efficiency means greater potential opportunity for profitability. Choice "II" - Whenever money is exchanged internationally, exchange rate risk exists. Choice "III" - Anytime correlation is less than 1, risk is reduced. Choice "IV" - Tax on capital gains must be paid by U.S. citizens, even in foreign investments.

Randal was just hired by Chastain, Inc., which sponsors a defined benefit plan. After speaking with the benefits coordinator, Randal is still confused regarding eligibility and coverage for the plan. Which of the following is correct? The plan could provide that employees be age 26 and have 1 year of service before becoming eligible if upon entering the plan, the employee is fully (100%) vested. The plan may not cover Randal due to his position in the company, even if Randal meets the eligibility requirements. Part-time employees, those that work less than 1,000 hours within a twelve-month period, are always excluded from defined benefit plans. Generally, employees begin accruing benefits as soon as they meet the eligibility requirements.

The correct answer is B. Choice a is not correct because the general eligibility is age 21, not 26. Choice c is not correct because a plan could cover part time employees, but will generally not. Choice d is not correct because employees become part of a plan only as early as at the next available entrance date after meeting the eligibility requirements.

Kipton is an executive with BigRock. As part of his compensation, he receives 10,000 shares of restricted stock today worth $20 per share. The shares vest two years from today, at which point the stock is worth $30 per share. The vesting schedule is a 2-year cliff schedule. Kipton holds the stock for an additional 18 months and sells at $45 per share. Which of the following is correct? The grant of stock is taxable to Kipton today. The value of the shares is taxable to Kipton when the stock vests. If Kipton were to make an 83(b) election, he would have converted $30 of the gain from ordinary to capital. When Kipton sells the stock for $45 per share, his basis is $30 regardless of whether he files an 83(b) election.

The correct answer is B. Choice a is not correct because the stock is forfeitable. Choice c is not correct because it would have converted $10, not $30. Choice d is not correct, because the basis would be different.

Which of the following statements are true in regards to Section 457 plans? Eligible plan sponsors include non-profit organizations, churches, and governmental entities. In-service distributions after age 59 1/2 are allowed in a 457 plan. Salary deferrals are subject to Social Security, Medicare, and Federal unemployment tax in the year of the deferral. Assets of the plans for non-government entities are subject to the claims of the sponsor's general creditors. I and III only. II, III and IV only. I, II and IV only. III and IV only.

The correct answer is B. Churches are not qualifying sponsors of 457 plans.

Byron is 46 years old and works for two employers, earning salaries of $52,000 and $48,000 respectively. One of his employers sponsors a 401(k) plan, and the other employer sponsors a 457(b) plan. For 2020, what is the maximum pre-tax elective deferral Byron can make in total to the two plans? A. $19,500. B. $39,000. C. $49,000. D. $57,000.

The correct answer is B. Contributions to a Section 457(b) plan do not count against the 401(k) plan limit. Therefore, Byron can contribute the maximum to each plan in the same year. For 2020, the contribution limit to each plan is $19,500, therefore, Byron can contribute at total of $39,000 ($19,500 + $19,500).

Mutual fund XYZ has a beta of 1.5, standard deviation of 12% and a correlation to the S&P 500 of .80. How much return of fund XYZ is due to the S&P 500? 20%. 64%. 80%. 100%.

The correct answer is B. Correlation is .80, therefore r-squared is .64 (R-squared = correlation coefficient squared). Therefore 64% of mutual fund's return is due to the S&P 500. Remember, r-squared measures the percentage of return due to the market.

Tom operates an illegal drug operation and incurred the following expenses: Salaries = $50,000; Illegal kickbacks = $20,000; Bribes to border guards = $25,000; Cost of goods sold = $150,000; Rent = $8,000; Interest = $10,000; Taxable income = $400,000. How much is his taxable income reduced by, based on the above expenses? $-0- $150,000 $218,000 $263,000

The correct answer is B. Cost of goods sold is the only deduction allowed for illegal drug operation activities.

Donald and Daisy are married and file jointly. They are both age 42, both work, and their combined AGI is $115,000. This year Donald's profit sharing account earned over $5,000. Neither he nor the company made any contributions and there were no forfeitures. Daisy declined to participate in her company's defined benefit plan because she wants to contribute to and manage her own retirement money. (Her benefit at age 65 under the plan is $240 a month.) How much of their $12,000 IRA contribution can they deduct? Assume that $6,000 is contributed to each account. $6,000 $8,700 $9,600 $12,000

The correct answer is B. Daisy is an active participant. She cannot opt out of a defined benefit plan. Reduction = 6,000 × [(115,000-104,000) ÷ 20,000] Reduction = 3,300 $6,000 - $3,300 = $2,700 deductible contribution. Donald is not active in the current year so he is eligible for a spousal IRA of $6,000 $6,000 + $2,700= $8,700 Formula: Contribution × [(your AGI - the bottom of the phase out range) divided by the amount of the range (124,000-104,000 is where the 20k comes from)]

Walt purchased a $1,000 par value bond that matures in 7 years. The bond has a current yield to maturity of 8%, and has a duration of 5.85 years. Walt believes that market interest rates will decrease by 1% in the future. If Walt's forecast is accurate, his bond investment will increase by: A. 4.43%. B. 5.42%. C. 5.67%. D. 5.85%.

The correct answer is B. Duration can be used to solve for a change in bond price when interest rates change. The formula to determine the change in bond price is: Change in bond price = -[Duration / (1 + y)] x Change in y Change in bond price = -[5.85 / (1.08)] x -.01 Change in bond price = -5.42 x -.01 Change in bond price = 5.42%

Dina loans $24,000 to her daughter Erin and does not charge any interest. Erin has investment income of $1,400 and investment expenses of $300. The applicable federal rate is 5%. How much interest must be imputed on the loan? $1,000. $1,100. $1,200. $1,400.

The correct answer is B. Erin has net investment income of $1,100. Therefore, the amount of imputed interest is the lesser of net investment income or interest calculated using the AFR less interest calculated using the stated rate of the loan. Since the stated rate of interest on the loan is 0%, the amount of imputed interest is the lesser of $1,100 or $1,200 ($24,000 × 0.05). Therefore, $1,100 of interest must be imputed on the loan.

Which of the following accurately describes a qualified group life insurance plan? The plan must benefit 70% of all employees, or a group consisting of 85% non-key employees, or a non-discriminatory class, or meet the non-discrimination rules of Section 125. Employees who can be excluded are: those with fewer than 3 years service, part-time / seasonal, non-resident aliens, or those covered under a collective bargaining unit. A non-discriminatory classification is one which has a bottom tier with benefits no less than 10% of the top tier and no more than 200% increase between tiers. The minimum group size is 10. I, II and III only. I, II and IV only. I and III only. I and II only.

The correct answer is B. For statement III to be a correct choice, it should state: A qualified group life insurance plan, if using a non-discriminatory classification, will have a bottom tier with benefits no less than 10% of the top tier and no more than 250% increase between tiers.

A qualified money purchase pension plan contribution (by the employer) is influenced by which of the following factors: Total return on portfolio assets. Forfeitures from non-vested amounts of terminated employee accounts. Increases in participants' compensation due to inflation or performance-based bonuses. Minimum funding as determined by an actuary. I and II only. II and III only. III and IV only. II, III and IV only

The correct answer is B. Forfeitures may reduce employer contributions due to contribution offsets or Section 415 limitation on annual additions. Increased compensation will result in increased contributions by the employer, subject to Section 415 limitations. Returns on portfolio assets and actuary funding are a concern in defined benefit plans. A money purchase plan is defined contribution plan.

Defined benefit pension plans increase the funding costs associated with the plan if the following actuarial assumptions are adjusted as follows: Turnover rate is reduced. Retirement age is reduced. Rate of Return is Increased. Wages Increase. I and III only. I, II and IV only. II and III only. II, III and IV only.

The correct answer is B. If turnover is reduced, then funding will increase. If the retirement age is reduced, funding will increase as more years will need to be funded. If the rate of return increases, then funding will decrease. If wages increase, then funding will increase.

In order to deduct a contribution to an IRA, which of the following requirements must be met? I. An individual must have earned income, either personally or jointly from a spouse. II. Must not be an active participant in an employer-sponsored qualified plan. III. Must be under the age of 70 1/2. IV. Must make contributions during the tax year or up to the date of filing the federal tax return for the tax year, including extensions. I and II only. I only. II and III only. IV only.

The correct answer is B. In 2020, contributions are limited to the lesser of 100% of earned income or $6,000 or $7,000 if age 50 or over. Deductions may be taken even if an active participant, so being a non-participant is not a requirement. There are no age restrictions to make contributions to an IRA; the taxpayer must have earned income (SECURE Act 2019). Contributions must be made prior to April 15 (or the mandated filing date for the year.) No extension to make the contribution is allowed after that date, even though an extension to file the return is granted. You CAN be an active participant, but as such you are subject to AGI deduction phaseout. So it's active participant PLUS AGI that may restrict your deduction.

Which of the following would cover the cost of a hospital stay in excess of 100 days? A. Medicare. B. Medicaid. C. Indemnity insurance policy. D. Medicare supplement.

The correct answer is B. Individuals have three methods to pay for stays after the 100th day: 1. Long-term care insurance 2. Individual savings 3. Medicaid

Sylvia has a two assets in her portfolio, asset A and asset B. Asset A has a standard deviation of 40% and asset B has a standard deviation of 20%. 50% of her portfolio is invested in asset A and 50% is invested in asset B. The correlation for asset A and asset B is .90. What is the standard deviation of her portfolio? Greater than 30%. Less than 30%. Equal to 30%. Not enough information to determine.

The correct answer is B. It's not necessary to use the standard deviation of a two asset portfolio formula to answer this question. Since there's a 50/50 weighting for each asset, simply take a simple average of the standard deviations (.40 + .20) / 2 = .30. Since the correlation is less than 1, the standard deviation for the portfolio will be less than the simple average. If correlation was equal to 1, then the standard deviation would be equal to 30%.

Under the basic approaches commonly in use in the no-fault auto insurance dilemma, which of the following best describes the plan where injured parties do not give up the right to sue, but simply refrain from such action until either a dollar threshold or a verbal threshold is reached? Extended first party coverage. Modified no-fault coverage. Pure no-fault coverage. Unsatisfied judgment no-fault coverage.

The correct answer is B. Modified no-fault coverage is the plan where injured parties do not give up the right to sue, but simply refrain from such action until either a dollar threshold or a verbal threshold is reached.

Shawnte has AGI of $1,000,000 (which is all comprised of earned income). She is single and age 55. She participates in her employer's 457 plan. Which of the following statements is true? She can contribute $6,000 to a Traditional IRA and deduct all $6,000. She can contribute $7,000 to a Traditional IRA and deduct all $7,000. She can contribute $6,000 to a Traditional IRA and deduct $0. She can contribute $7,000 to a Traditional IRA and deduct $0.

The correct answer is B. Participating in a 457 plan is not considered being an "Active Participant." She can contribute and deduct her contribution to a Traditional IRA $6,000 (2020) since she is not an active participant and therefore not subject to an AGI limitation. She is unable to contribute to a Roth IRA because she is above the AGI limitation of $124,000 - $139,000 (2020). Because she is 50 or older she is allowed to make the $1,000 (2020) catch up contribution.

Cher, who just turned 57 years old, took early retirement so she could spend more time with her three grandchildren and to work on her golf game. She has the following accounts: 401(k) Roth account - she has a balance of $100,000. She only worked for the company for four years and contributed $15,000 each year to the Roth account. The company never contributed anything to her account. Roth IRA - she has a balance of $80,000. She first established the account by converting her traditional IRA ($50,000 all pretax) to the Roth IRA 4 years ago and has contributed $5,000 each of the last 4 years. Cher decided that she would take a distribution of half of each account ($50,000 from the Roth 401(k) and $40,000 from the Roth IRA) for the purpose of purchasing a Porsche Cayenne, which of course would be used to carry her new Ping golf clubs. Which of the following is correct regarding the tax treatment of her distributions? No tax, no penalty on either distribution. Taxation on $20,000 from the 401(k) Roth and a penalty on $20,000 from the Roth IRA. No taxation on the distribution from the 401(k) Roth, but income and penalty on $20,000 from the Roth IRA. Penalty of $2,000 on the Roth distribution and taxation and penalty on $20,000 of the Roth 401(k) distribution.

The correct answer is B. Neither distribution is qualified. Non-qualified distributions from a Roth account consist of basis and earnings on a pro rata basis. Therefore, 60% of the Roth account distribution is return of basis. The remaining 40% or $20,000 is subject to income tax. Because the distribution is from a qualified plan and she has separated after the attainment of age 55, there is no penalty. Non-qualified distributions from a Roth IRA come out in the order of contributions, conversions and then earnings. The first $20,000 is not subject to income tax or penalty because it is from contributions. The second $20,000 is from conversions, which have been subject to taxation. However, because she rolled them over within the last five years, she will have a penalty and there is no exception Additional detail: There are two distributions, and each has slightly different rules. Roth 401k Balance: 100,000 of which: Contributions: 60,000 Earnings: 40,000 Distribution after 4 years, and separation of service: 50,000 Roth 401k is a pro-rata tax on distributions. 60% is return of contributions 40% is earnings The distribution of 50,000 has 40% of it subject to tax, which is 20,000. No early withdrawal penalty due to separation of service after age 55. Roth IRA Balance 80,000 of which: Conversion: 50,000 Contributions: 20,000 Earnings: 10,000 The Roth has an order to distributions, contributions (tax and penalty free), conversions, and then earnings. The $40,000 distributions breaks down as: 20,000 of contributions tax and penalty free 20,000 from conversion which is tax free (paid tax at conversion) and is subject to 10% early withdrawal penalty (under age 59 ½) In summary: 20,000 is taxed from Roth 401k, and 20,000 from the Roth IRA is subject to the 10% penalty

Ashton, age 29, purchased stock in a real estate investment trust (REIT) with a dividend yield of 9%. He was concerned that the real estate market may have been hurt in the recent market turmoil. However, they kept paying the stated dividend but his 1099-DIV only lists 4% as ordinary dividends. How can this be possible? The REIT was paying 5% in qualified dividends. The REIT was making nondividend distributions. The REIT was selling his shares. The REIT already paid tax on the other 5% so Ashton does not have to.

The correct answer is B. Nondividend distributions are a return of capital and are not included for gross income. (A) is incorrect because REITs cannot pay qualified dividends. (C) is incorrect because the REIT cannot sell an owner's shares. (D) is incorrect because the REIT cannot pay the tax for the shareholder.

One of your clients is 47 years old and wants to know the maximum amount which might be allocated to her 401(k) account in the current year. She expects to earn $60,000. You explain the possible sources of annual additions to her account, including allocation of forfeitures from departing non-vested employees, and the limitation on that addition. The largest salary deferral which could be made to her 401(k) account in the current year is: $13,500 $19,500 $26,000 $57,000

The correct answer is B. Note the legal limit for 2020 is $19,500 for employee deferrals to a 401K account. The $57,000 is the total limit for an employee under 50 from all sources including employee deferral, employer match, forfeitures, and profit sharing contributions by the employer.

Olive's daughter Polly suffers from a rare illness. During the current year, Olive drove Polly to see a specialist in another state 15 times, while her husband remained home running their software company. Each trip was 300 miles each way and required an overnight stay in a hotel that costs $140 per night. The unreimbursed treatments totaled $10,000. Olive's joint AGI is $124,000. How much can they deduct in relation to Polly's medical expenses for 2020? $9,300 $3,730 $300 $0

The correct answer is B. Olive may deduct $0.17 (2020) cents per mile for the travel associated with Polly's medical care and may deduct up to $50 per night per person for lodging. Therefore, the total medical expenses are $13,030 which is calculated as follows: [(300 x 2 x 15 x $0.17=1,530) + (15 x ($50 x 2)=1,500) + $10,000]. The trip mileage is stated as 300 miles each way, you will need to multiply by 2 and use round trip mileage.However, Olive may only deduct the amount that exceeds 7.5% of their AGI . 7.5% of Olive's AGI is $9,300. Therefore, medical expenses deductions for Polly's treatment is $3,730. Note, $100 is allowed because the rule is $50 per person per night

A financial planner's client has an IRA with a balance of $140,000 as of January 1. On April 15 of the same year, the client withdraws the entire amount from the IRA and places it in a non-IRA CD for 60 days, earning 9% interest. On the 60th day, the client promptly and timely reinvests the principal of the CD in an IRA containing an aggressive growth fund. On September 15 of the same year, the client becomes dissatisfied with the return and the variability of the investment. The client wants a less risky investment and wants assurance that any IRA distribution will NOT be taxed at the time of the change. Which of the following is/are acceptable alternatives for the client? Withdraw the funds and reinvest them within 60 days in an IRA which invests exclusively in Treasury instruments. Direct the trustee of the IRA to transfer the funds to another IRA which invests exclusively in Treasury instruments. Withdraw the funds and reinvest within 60 days in an IRA which is an index mutual fund holding common stocks with portfolio risk equal to the S&P 500. I only. II only. II and III only. I, II and III.

The correct answer is B. Once an individual has participated in a rollover where funds have been withdrawn and held and then reinvested, he or she is ineligible for such a transaction again for one year from the date of receipt of the amount withdrawn. In this question, Statement "II" is the only choice which prevents penalty to the client.

Which of the following is false regarding incentive stock options? No regular taxable income will be recognized by the employee when the qualified option is granted or exercised. The income from sale of the qualified option will always be taxed as capital gains when the stock is sold. The income from sale of the qualified option will be taxed as ordinary income regardless of when the stock is sold. The employer will not be able to deduct the bargain element of the option as an expense under any circumstance. For favorable tax treatment the stock must be held two years from grant and one year after exercise. II and IV only. II, III and IV only. III only. I, II and V only.

The correct answer is B. Only Statements "I" and "V" are true. The rest are all false. In Statement "II", be careful of "always"! In Statement "III," if held longer than one year, they receive capital gains treatment. In Statement "IV," the bargain element will be deductible if the sale is a disqualifying disposition.

Defined benefit pension plans will have to increase the funding costs associated with the plan if which of the following actuarial assumptions are made: Low turnover rate. Early retirement. High interest rate. Late retirement. Salary scale assumption. I and III only. I, II and V only. I and V only. II, III and V.

The correct answer is B. Options I, II and V increase funding costs of a DB plan.

All of the following are acceptable reasons for an employer to terminate a qualified retirement plan except: The employer is no longer in a financial position to make further plan contributions. The employer no longer wants to maintain the plan because it must cover other employees other than just himself. The plan benefits are not meaningful amounts, and participants are limited in their ability to make deductible IRA contributions. To lower plan costs and ease administrative complexity, the employer wants to switch plan designs.

The correct answer is B. Retirement plans must not be created as a tax shelter for the owner. If they have been, plan termination can result in retroactive disqualification. All other statements are acceptable reasons to terminate a qualified retirement plan.

Risk transfer (or the use of insurance) would best be utilized in the case of: High frequency / high severity. Low frequency / high severity. High frequency / low severity. Low frequency / low severity.

The correct answer is B. Risk transfer (or the use of insurance) would be utilized in the case of low frequency / high severity. We use insurance to transfer risk for life, income and property. All areas we hope to have no losses, but if we did, they would alter our way of living. Low FrequencyHigh FrequencyLow SeverityRetentionReduction.Non-Insurance TransferHigh SeverityInsuranceAvoidance (Catastrophic risk)

Satish has AGI of $66,000 (which is all comprised of earned income). She is single and age 51. Her employer made a $6,300 contribution to her SEP for the current year. What is her available deduction allowed for a Traditional IRA contribution? $26,000 $6,300 $6,000 $7,000

The correct answer is B. She can contribute to a Traditional IRA since she has earned income. She is considered an active participant because her employer made a contribution to the SEP on her behalf. Her deduction will be limited because she is within the AGI limitation for a single active participant ($65,000 - $75,000) (2020). She is also entitled to the catch up contribution of $1,000 because she is 50 or older. Therefore, her deductible contribution is: 7,000 × ((66,000 - 65,000)/10,000) = $700 is not allowed so $7,000 - $700 = $6,300 is permitted. Although the question didn't ask - alternatively, she could contribute to a Roth IRA because she is below the AGI limitation of $124,000 - $139,000 (2020).

Scary Berries, Inc. is a C corporation that specializes in carving berries into frightening images. The company's primary profit generation is the sale of berries and they generate $4 million in annual revenue on average. What accounting method may Scary Berries, Inc. utilize for tax purposes? Cash basis Accrual basis Either cash or accrual Units of production

The correct answer is B. Since Scary Berries generates its income from inventory (the berries) they must file under the accrual basis even though their revenues are under the $25 million accrual basis.

Tammy, age 62, received $32,000 of rental income from various rental properties she owns. This was her only source of income for the year. What is the best investment strategy for her retirement? A. SEP. B. Brokerage account. C. Traditional IRA. D. Safe-harbor 401(k) plan

The correct answer is B. Since rental income is not considered earned income, Tammy is not eligible to contribute to a traditional or Roth IRA. She also cannot contribute to a 401(k) plan.

Which statement(s) accurately reflect(s) the Tax-Sheltered Annuity (TSA) provisions: Salary reductions into a TSA are exempt from all payroll taxes. The annual elective deferral limit may be increased by up to $3,000 for employees of certain organizations who have completed 15 years of service and meet certain other requirements. Tax sheltered annuities must allow participants to invest in mutual fund, annuities and/or fixed income securities. To calculate the maximum exclusion allowance for make-up calculation purposes, the participant's years of service and the amount of total excludable contributions made in the prior three years are needed. I and II only. II only. I, III and IV only. IV only.

The correct answer is B. Statement "I" is incorrect because deferrals are still subject to Social Security and Medicare taxes. Statement "III" is incorrect because TSAs can only invest in mutual funds or annuities and not any direct investments. Statement "IV" is incorrect because the total excludable contributions must be for all prior years, not just the past three.

Which of the following statements concerning rabbi trusts is (are) CORRECT? A rabbi trust is a trust established and sometimes funded by the employer that is subject to the claims of the employer's creditors, but any funds in the trust cannot generally be used by or revert back to the employer. A rabbi trust calls for an irrevocable contribution from the employer to finance benefits promised under a nonqualified plan, and funds held within the trust cannot be reached by the employer's creditors. A rabbi trust may not be held off-shore as a result of the American Jobs Creation Act of 2004. The American Jobs Creation Act of 2004 prohibits "springing irrevocability" for a rabbi trust if there is a change of control or ownership. I and IV only. I and III only. II and III only. I only.

The correct answer is B. Statement "II" describes a secular trust. Statement "IV" is incorrect because AJCA 2004 does allow springing irrevocability in these circumstances, but not for bankruptcy.

Which of the following statements apply to distributions made from Individual Retirement Accounts (IRA)? I. Distributions to the IRA owner must begin by April 1 of the year following the year in which the owner reaches age 70 1/2 (if by 12/31/2019) or age 72 (if 70 1/2 after 12/31/2019). II. If funds in a rollover IRA (originated in an employer-sponsored qualified retirement plan) are not "tainted" with other contributions, the distribution may be eligible for 5-year forward averaging tax treatment. III. After the owner's death, the entire amount remaining in the IRA is included in the owner's gross estate for federal estate tax purposes. IV. Distributions taken prior to age 59 1/2 may be exempt from penalty only if the owner separated from service after age 55 and the original plan document allowed early retirement at age 55. I and II only. I and III only. II and IV only. I, III and IV only.

The correct answer is B. Statement "II" is incorrect because funds distributed from an IRA are always treated as ordinary income, regardless of source and 5 year forward averaging is no longer available for any distribution. Statement "IV" is incorrect because all distributions from an IRA not meeting the statutory exemptions are subject to the premature distribution penalty, regardless of source. *always slow down and critically examine your options.

Financial Training Team (FTT) develops training materials for finance professionals across the country. Chad, who just turned age 48, owns 15% of FTT and earns $200,000 per year and is a participant in his employer's 401(k) plan, which includes a qualified automatic contribution arrangement and the associated mandatory non-elective contribution. The actual deferral percentage test for the non-highly compensated employees is 2.5 percent. FTT made a 20% profit sharing plan contribution during the year to Chad's account. What is the maximum amount that Chad can defer in the 401(k) plan during 2020? $26,000 $11,000 $13,500 $19,500

The correct answer is B. The 401(k) plan avoids ADP testing because it is a QACA. Therefore, the ADP for the NHCE is irrelevant. However, the max that can be contributed is limited by IRC 415(c). The employer is contributing $40,000 to the profit sharing plan plus $6,000 as a non-elective contribution (3% of $200,000). Since the 2020 limit is $57,000, Chad can only contribute $11,000.

The HMO model under which the subscribers have the greatest flexibility is: The staffing model. The IPA model. The group model. The network model.

The correct answer is B. The IPA (Individual Practice Association) allows the greatest flexibility among HMO coverages.

When Pete Morin purchased his $100,000 home, he insured it at the required coinsurance amount of 80% of the value. Over the last five years, the value of his home has increased and is now $160,000, but he has not increased his coverage. Pete has a $500 deductible. He has a kitchen fire causing $10,000 in damage. What amount will his insurer pay for repairs? $4,250 $5,750 $6,250 $9,500

The correct answer is B. The amount carried divided by the amount required (80% of current value) times the loss, minus the deductible equals the payment. One of the tricks on this one is that he purchased $80,000 of coverage initially (80% of the purchase price). So, the covered loss equals [$80,000 / (.80 × $160,000)] × $10,000 = $6,250. The insurer will pay $6,250 - $500 = $5,750.

Lauren has purchased a home worth $1.5 million with an interest-only mortgage of $1.2 million on 12/20/17. She is currently only paying interest on the mortgage in the amount of $60,000 per year. What amount may she deduct as home mortgage interest on Schedule A of her individual income tax return? $0 $37,500 $55,000 $60,000

The correct answer is B. The calculation is calculated by dividing the qualified mortgage over the total mortgage times the interest paid. (750,000/1,200,000) × 60,000 = 37,500 Per TCJA, home mortgages are limited to qualified residential interest and a maximum indebtedness of $750,000 if financed after 12/15/17. (For debt prior to 12/15/17, the $1 million limit applies.)

You have been hired to analyze the retirement prospects of Tom and Jerri Ruhn. It has been determined they need a retirement capital account of $2,750,000 at retirement which will occur in 30 years. They expect to live in retirement for 35 years. They are anxious to start a savings program to meet this goal. They anticipate an average after-tax rate of return equal to 7%. They are planning on 5% annual inflation. What level of savings put away at the end of each year will provide the Ruhn family with their desired retirement fund? $27,208 $29,113 $67,787 $68,884

The correct answer is B. The client has given us the capital account they want at retirement in 30 years. If it said in today's dollars or the equivalent, then you would account for inflation. That information was a distractor in this question. N = 30 I = 7 PV = 0 PMT = 29,113 FV = 2,750,000

The investment portfolio for a defined-benefit retirement plan has declined in value during a year in which most financial market instruments have also incurred losses. Which one of the following entities would be impacted most by this decline in portfolio value? Individual participants in the plan. Company sponsoring the plan. Investment advisory handling the plan assets. Plan underwriters.

The correct answer is B. The company may be required to make increased contributions to fully fund the plan.

Evaluate the following statements: De minimis fringe benefits are those that are so immaterial that . . . De minimis fringe benefits are subject to strict anti-discrimination . . . I and II are true. I is true, but II is false. I and II are false. I is false, but II is true

The correct answer is B. The de minimis exemption is not subject to a non-discrimination requirement because the amounts are too small to make it worthwhile to account for the items.

For purposes of the Social Security earnings test, which of the following types of income would potentially cause a reduction in Social Security benefits? A. Dividends received from a global mutual fund. B. Royalties from a published novel. C. Distribution from a defined benefit plan. D. Required minimum distribution received from a traditional IRA.

The correct answer is B. The earnings test only applies to earned income received by the Social Security recipient. A is incorrect. Dividends received from investments are considered unearned income. C is incorrect. Distributions from retirement plans are considered unearned income. D is incorrect. Distributions from retirement plans are considered unearned income.

Which of the following penalties are assessed when prohibited transactions occur? 10% of the amount involved unless shown that ERISA fiduciary standards were satisfied. Penalties can continue when ongoing transactions carry over to subsequent years. The plan must be restored to a financial position no worse than if the transaction had never occurred. Income tax will be assessed against those plan participants who were party to the transaction by the courts. I and III only. II and III only. III and IV only. I and II only.

The correct answer is B. The first tier excise tax for prohibited transactions is 15% of the amount involved and is automatic even if the violation was inadvertent. The second tier excise tax is 100% of the amount involved and is assessed if the prohibited transaction is not remedied. There are no income taxes applied; all remedies are made through restitution and excise taxes.

Courtney's portfolio has a covariance with the S&P 500 of 0.01408. The standard deviation of the S&P 500 is 16 percent. Calculate the beta of Courtney's portfolio. .50 .55 .85 1.15

The correct answer is B. The formula for Beta is on the CFP Board provided Formula Sheet and in your Investment pre-study book. This formula is unusual as you can use either portion of it (the formulas will be equal). Since we are provided the COV, we can use the left side of the formula, as both sides will be equal. The first part would look like this: .01408 .162 And will equal .55

An analysis of the monthly returns for the past year of a mutual fund portfolio consisting of two funds revealed the following statistics: Fund A total return = 18% Fund A Standard deviation = 23% Fund A Percentage of portfolio = 35% Fund B total return = 11% Fund B Standard deviation = 16% Fund B Percentage of portfolio = 65% The Correlation Coefficient ("R") between the two funds equals .25. What is the standard deviation of the portfolio? 13.16% 14.66% 18.45% 19.50%

The correct answer is B. The information in [ ] is the COV formula. compute that prior to multiplying 2(.35)(.65)COVij s p = √(.35)2(.23)2 + (.65)2(.16)2 + 2(.35)(.65)[(.23)(.16)(.25)] s p = √.0065 + .0108 + .0042 s p = √.0215 s p = .1466

Which branch of government is charged with interpreting and enforcing state insurance code rulings that have the force of law? Judicial. Executive. Legislative. Public Safety.

The correct answer is B. The key word here is "enforcing." That is what the executive branch does. Had the question asked about interpreting and rendering opinions, it would have been referring to the state judicial branch.

James is covered under his employer's top heavy Defined Benefit Pension Plan. He currently earns $120,000 per year. The Defined Benefit Plan uses a funding formula of Years of Service × Average of Three Highest Years of Compensation × 1.5%. He has been with the employer for 5 years. What is the maximum defined benefit that can be used for him for funding purposes? $9,000 $12,000 $54,000 $120,000

The correct answer is B. The maximum defined benefit is the lesser of $230,000 (2020) or his compensation. However, the funding formula will limit his defined benefit to $12,000 (5 × 120,000 × .02). Note that you would use 2% instead of the 1.5% because the plan is top heavy. He is not a key employee because he is not a 1) greater than 5% owner, 2) greater than 1% owner with compensation greater than $150,000 or 2) an officer with compensation greater than $185,000 (2020). Therefore the plan must use a defined benefit limit of 2% instead of 1.5%.

Richard is covered under his employer's Defined Benefit Pension Plan. He earns $200,000 per year. The Defined Benefit Plan uses a funding formula of Years of Service × Average of Three Highest Years of Compensation × 3%. He has been with the employer for 25 years. What is the maximum defined benefit that can currently be used to determine contributions? $57,000 $150,000 $230,000 $285,000

The correct answer is B. The maximum defined benefit is the lesser of $230,000 (2020) or his compensation. However, the funding formula will limit his defined benefit to $150,000 (25 × 200,000 × .03).

Which of the following can be eliminated using a "buy and hold" strategy with regard to fixed income securities? Future value risk. Interest rate risk. Stand alone risk. Reinvestment rate risk

The correct answer is B. The price changes when interest rates change but if you don't sell the bond (buy and hold) then the price change doesn't really matter. You still get the $1,000 par value at maturity. Option "A" - Future value risk does not exist as a term. Option "C" - Stand alone risk refers to single assets ownership. Option "D" will still require the investor to reinvest interest paid, thus not eliminating such risk.

George is analyzing his investment portfolio. He has an investment in the XYZ Mutual Fund (the fund). Historical returns of the fund, as well as Treasury bills, are as follows: XYZ Mutual Fund Treasury bills Year 1 4% 4% Year 2 -6% 1% Year 3 -6% 3% Year 4 12% 2% Year 5 -4% 5% What is the Sharpe ratio of XYZ Mutual fund? A. -0.423 B. -0.381 C. 0.000 D. 1.3970

The correct answer is B. The risk-free rate (average Treasury Bill rate) is 3%, calculated as follows: Average risk-free rate = (4% + 1% + 3% + 2% + 5%) / 5 Years Average risk-free rate = (15%) / 5 =3% The average rate of return on the XYZ Mutual Fund is 0%, calculated as follows: Average return on XYZ Mutual Fund = (4% -6% -6% +12% -4%) / 5 Years Average return on XYZ Mutual Fund = (0%) / 5 Years =0% The standard deviation of XYZ Mutual Fund is 7.8740, calculated as follows: Standard Deviation (each rate of return should be entered in the calculator, followed by the "sigma +" key)= 4, Σ+ 6+/-, Σ+ 6+/-, Σ+ 12, Σ+ 4+/-, Σ+ Press the [SHIFT] 8 on a HP10BII+ calculator (SxSy) Sharpe Ratio is (Average Return - Risk-Free Rate)/ Standard Deviation Sharpe Ratio = (0%-3%)/(7.8740) = -0.381

The Federal Reserve Board is expected to sell large quantities of Treasury securities in the near future. What impact will these sales likely have on stock prices? Stock prices will decrease because the dividend growth rate of stocks will increase. Stock prices will decrease because the required rate of return for investors will increase. Stock prices will increase because interest rates will decrease as investors compete to purchase the Treasury securities. Stock prices will increase because the growth rate in dividends and earnings will increase.

The correct answer is B. The sale of Treasury securities results in a reduction of cash in the market place, thus a decrease in supply causing an increase in demand. This will lead to an increase in the cost of money and a lessening of funds for investment, thus a reduction in stock prices.

As the fiduciary for a company defined benefit plan, you were recently approached by the employer, a sole proprietor. She requested a loan to the company from the assets of the pension plan to purchase equipment needed by the company. You respond that the loan request is: Acceptable if made to the employer as a participant in the plan. Acceptable if it bears a reasonable rate of interest in respect to current market rates. Acceptable since the money will be invested in the company and is NOT intended to directly benefit the employer. NOT acceptable, since the employer is considered a party-in-interest. II only. IV only. I and II only. I, II and III only.

The correct answer is B. This is an example of a prohibited transaction. PLAN LOANS PERMITTED TO S CORP SHAREHOLDERS.

June and Bud, both 40 years old, are not covered by a qualified retirement plan. Bud, trying to maximize their IRA deduction, put $12,000 into an IRA with June as the beneficiary on December 15 of the current year. What best describes the result of this transaction? June and Bud receive a tax deduction for the entire $12,000 because both spouses are eligible to contribute $6,000 to the IRA. Bud receives a tax deduction for $6,000 and a 6% penalty for over-contribution on the other $6,000. Next year Bud will receive a $6,000 deduction, in addition to the $6,000 deduction for this year. Bud receives a tax deduction for $6,000 and is considered to have made a non-deductible contribution of the other $6,000.

The correct answer is B. This question indicates an IRA in only Bud's name. Maximum contribution is $6,000 plus any applicable catch-up provisions. Amounts contributed over that level are considered excess contributions and subject to a 6% penalty until taken out. The 6% penalty could have been avoided if the excess contribution was withdrawn prior to the original filing deadline without extension.

Jerry Rivers owns a $250,000 level-term life policy which he purchased five years ago. He has paid premiums of $400 per year for the past five years. He also owns a $125,000 whole life policy which he purchased fifteen years ago. He has paid premiums of $2,000 per year for the past 15 years, and now the policy has a cash surrender value of $40,000. Over the years, the whole life policy has paid cash dividends to Jerry. The cumulative dividends paid to Jerry since inception totals $5,000. Jerry has decided to cancel his $125,000 whole life policy. Which statement is true? Jerry has a taxable gain of $10,000. This gain will be treated as a long-term capital gain. Jerry has a taxable gain of $15,000. This gain will be treated as ordinary income. Jerry has a taxable gain of $70,000. This gain will be treated as ordinary income. Jerry has NO taxable gain.

The correct answer is B. Upon surrendering his whole life policy, Jerry received $40,000 cash value where he had paid only $30,000 - $5,000 (dividends) = $25,000 (basis). $40,000 - $25,000 = $15,000 is treated as ordinary income, taxable in the year it is received.

Meredith is an executive at Papers Unlimited. As part of her compensation she has a restricted stock plan that allows her to receive 1,000 shares of stock after she completes of 5 years of service. At the time of grant the stock was trading at $2 per share. She made a proper 83b election. She met the vesting requirement 6 months ago when the stock was trading at $35. She has decided to sell her stock. Which of the following is true? If she sells the stock today for $1 per share she is not entitled to a loss. If she sells the stock today for $15 per share she will recognize $13,000 in long term capital gain. If she sells the stock today for $28 per share then she will recognize $26,000 in short term capital gains. If she sells the stock today for $38 per share then she will recognize $3,000 in short term capital gains.

The correct answer is B. When she made the 83b election she would have recognized W-2 income of $2,000 (1,000 × $2). Her holding period would have started at the date of grant. When she met the vesting period she would not have recognized anything since she made the 83b election. If she sold the stock at $1 then she would have had a loss of $1,000 ($2,000 basis - $1,000 sale price). If she sold the stock for $15 then she would have long term capital gain of $13,000 (1,000 × ($15 - $2)). If she sold the stock for $28 then she would have long term capital gain of $26,000 (1,000 × ($28 - $2)). If she sold the stock for $38 then she would have long term capital gain of $36,000 (1,000 × ($38 - $2)).

Seema owns a landscaping company in Phoenix and wants a low-cost retirement plan that permits her employees to make pre-tax contributions. She is planning on contributing 3% of eligible employee's pay each year. Which of the following plan's would be most appropriate plan for Seema to establish? A. 401(k) plan. B. Profit sharing plan. C. SIMPLE IRA. D. SIMPLE 401(k) plan.

The correct answer is C. A SIMPLE IRA would permit employees to make pre-tax contributions. Also, the plan is low cost, and the 3% contribution Seema is planning to make would satisfy the employer matching contribution requirements of a SIMPLE plan. As qualified plans, A, B & D will all have higher cost structures than a SIMPLE IRA and therefore do not meet Seema's requirement of a low-cost plan.

Robin just started at Financial University Network (FUN) and has been encouraged by several of the "old timers" to save part of her salary into the 401(k) plan. She is not yet convinced as she likes to shop. Which of the following statements is accurate regarding 401(k) plans? A 401(k) plan must allow participants to direct their investments. Deferrals into the 401(k) plan must be contributed by the end of the following calendar quarter into the plan. Employees that join the plan must be provided with a summary plan description. A 401(k) plan is financially safe because it must have an annual audit.

The correct answer is C. Answer c is correct as employees must be given a summary plan description, which provides basic information about the operation of the plan. Answer a is not correct as some 401(k) plans may have the asset managed by an investment manager. However, most 401(k) plans will provide for employee self directing of their 401(k) balances. Answer d is not correct as there is no requirement for an annual audit of a 401(k) plan.

Match the following statement with the type of retirement plan which it most completely describes: "This plan can provide for voluntary participant contributions which must be matched by the employer." Profit sharing plan with a 401(k) component. Money purchase plan. SIMPLE IRA. Defined benefit plan.

The correct answer is C. Answers "B" and "D" do not permit employee elective deferrals. The profit sharing plan "A" with 401(k) provisions do not require an employer match.The SIMPLE plan has a mandatory match.

Your clients, Nick and Betty Jo Byoloski, have come to you with some questions. She has been an employee of April Corporation for several years and received some stock options as compensation at times. He has worked with April Corporation as a consultant on several jobs over the last few years and was paid in part with stock options. Nick and Betty Jo want to know more about their situation regarding the options. What can you tell them? Betty Jo's options are qualified and Nick's options are non-qualified. Nick's options are non-qualified and Betty Jo's options are non-qualified. Nick's options are non-qualified and Betty Jo's are either qualified or non-qualified. Betty Jo's options are non-qualified and Nick's options are qualified.

The correct answer is C. Because Nick is not an employee, we know that his options are non-qualified. We cannot be sure about Betty's without more information.

The following investment return will result in what dollar weighted return? An initial outlay of $50,000, with three years of additional outflows of $10,000 each, and inflows as follows: $0 the first year, $20,000 in years 2 and 3, and sale of the property at the end of year 3 for $75,000. 27.64% 14.04% 18.32% 20.67%

The correct answer is C. CF0 = <50,000> CF1 = 0 - 10,000 = <10,000> CF2 = 20,000 - 10,000 = 10,000 CF3 = 20,000 - 10,000 + 75,000 = 85,000 IRR = ?

Charlie purchased a 25% interest in a general partnership for $40,000. He is a material participant in the partnership's activities. In the current year, the partnership borrowed $20,000 from a local bank. The loan is considered a recourse loan. Assuming the partnership incurred a total loss for the current year of $200,000, how much of the loss will Charlie be permitted to deduct on his federal income tax return? A. $0. B. $40,000. C. $45,000. D. $50,000.

The correct answer is C. Charlie's share of the loss is $50,000 ($200,000 total loss x 25% ownership interest). However, his deductible loss is limited to the amount Charlie has "at risk." Charlie's "at risk" amount is equal to the amount he contributed to the partnership, plus his share of recourse loans incurred by the partnership. Charlie's share of recourse loans is $5,000 ($20,000 total loan x 25%). Therefore, the total amount at risk for Charlie is $45,000 ($40,000 contributed + $5,000 recourse loan).

Which of the following are true statements about the Capital Asset Pricing Model (CAPM)? The Security Market Line (SML) by itself does NOT determine the optimal portfolio for an investor. Beta is used as a measure of risk on the Security Market Line (SML). The required return is beta times the market return. As investors replace risk-free assets with risky assets, the required return will rise. I and III only. II and IV only. I, II, and IV only. II, III, and IV only.

The correct answer is C. Choice "III" - The required rate of return is determined by adding the risk premium (which is the market rate of return minus the risk free rate times the beta) to the risk free rate of return.

A premature distribution from a qualified retirement plan is allowed at age 52 without a 10% penalty tax when a participant: Becomes obligated for payment of plan benefits to an alternate payer under a qualified domestic relations order (QDRO). Separates from service and takes an accepted form of systematic payment. Remains with current employer but elects to take systematic payments over the life of the participant and spouse. I only. III only. I and II only. I, II and III.

The correct answer is C. Distributions under a QDRO are not taxable to the taxpayer actually making the disbursement from his/her account. IRC 72(t) allows Substantially Equal Payment Plans (SEPP) to escape the 10% penalty as long as the payments continue for the longer of 5 years or until age 59 1/2. No in-service withdrawals are exempted from the 10% early withdrawal penalty.

Your client, Dennis and Daughter, Inc. (often referred to as DAD by the owners) is a C corporation with gross revenues of $3,000,000 for the past four years. The net earnings to the firm for the most recent fiscal year were $120,000. There are two shareholders, Dennis and his daughter, Denise. They have recently had an outside consultant perform a valuation of the company using the capitalization method and a .10 capitalization rate. Based on this information, Dennis and Denise have decided to consider a buy-sell agreement. Using the above information, answer the following question. The value of the firm, as established by the outside consultant, is likely to be closest to: $833,333 $1,000,000 $1,200,000 $1,333,400

The correct answer is C. Divide the capitalization rate factor into the net earnings to arrive at the correct response. $120,000 ÷ .10 = $1,200,000

Dr. Woods, age 29, is a new professor at Public University (PU) where he has a salary of $111,000. PU sponsors a 403(b) plan and a 457 plan. Dr. Woods also has a consulting practice called Damage Estimate Claims (DEC). He generates $200,000 of revenue and has $50,000 of expenses for DEC. Assume his self-employment tax is $19,790. What is the most that he could contribute to all of the retirement plans this year assuming he establishes a Keogh plan for DEC? $38,000 $56,000 $67,021 $71,021

The correct answer is C. Dr. Woods can contribute $19,500 to each of the 403(b) plan and the 457 plan. In addition, he can establish a Keogh plan and contribute 20% of his net self-employment income after deducting ½ self,-employment taxes. The 403(b) and 457 can both receive 19,500 (qualified and deferred comp plans). The Keogh needs to follow self-employment contribution rules - see the retirement pre-study book) $200,000 of income for DEC -$50,000 of expenses for DEC 150,000 Net income -9,895 (Assume his self-employment tax is $19,790, use ½) 140,105 × 20% (contribution rate / (1+contribution rate) = self-employed contribution rate) 28,021 In total 19,500 + 19,500 + 28,021 = 67,021

According to ERISA, which of the following is/are required to be distributed automatically to defined benefit plan participants or beneficiaries? Annual accrued benefit as of the end of the previous year. The plan's summary annual report. A detailed descriptive list of investments in the plan's fund. Terminating employee's benefit statement. I, II and III only. I and II only. II and IV only. IV only.

The correct answer is C. Employee accrued benefits are established by the pension formula, therefore are not required to be provided each year. (Defined contribution plans must provide an annual statement of account.) Because the participant has no individual account in a DB plan, a detailed list of investments is not required. The participant in a DB plan generally has no input in the investment choices within the plan.

Which of the following statements concerning COBRA is correct? An employer's plan is exempt from COBRA provisions if the employer averages 25 or fewer employees. Continuation coverage must be available to terminating employees, but not to full-time employees shifting to part-time status. After 36 months, the maximum period for continuation of coverage terminates. The government imposes a non-compliance fine on the employer equal to $10 per day per participant

The correct answer is C. Employers must provide COBRA if they have 20 or more employees. A change in benefit status will trigger COBRA eligibility. COBRA non-compliance carries a penalty of $100 per day per participant.

Which of the following statements concerning the use of life insurance as an incidental benefit provided by a qualified retirement plan is (are) correct? The premiums paid for the life insurance policy within the qualified plan will trigger a taxable event for the participant at the time of payment. Under the 25 percent test, if term insurance or universal life is involved, the aggregate premiums paid for the policy cannot exceed 25 percent of the employer's aggregate contributions to the participant's account. If a whole life policy other than universal life is used, however, the aggregate premiums paid for the whole life policy cannot exceed 50 percent of the employer's aggregate contributions to the participant's account. In either case, the entire value of the life insurance contract must be converted into cash or periodic income at or before retirement. I. II. I and II. Neither I or II.

The correct answer is C. Every year the plan participant pays income tax on the dollar value of the actual insurance protection -- approximately equal to the term insurance cost. This is commonly called the PS58 cost. The sum of all those costs is the participant's basis.

Calculate the maximum contribution (both employer and employee elective deferrals) for an employee (age 39) earning $290,000 annually, working in a company with the following retirement plans: a 401(k) with no employer match and a money-purchase pension plan with an employer contribution equal to 12% of salary. $19,500 $34,200 $53,700 $57,000

The correct answer is C. For the purposes of this calculation, the compensation exceeding $285,000 is not recognized. The employer is contributing 12% of $285,000 (or $34,200) for the money purchase plan and the employee may contribute up to $19,500 in 2020 to the 401(k) plan. This totals $53,700.

The Chesapeake Bay apartment complex contains 60 one-bedroom apartments renting for $650 per month. In addition, the complex generates $625 per month from laundry, parking, and vending machines. Vacancy and collection losses have averaged 8% of Potential Gross Income (PGI) and are expected to continue at about the same rate in the future. Annual expenses totaling $117,000 include: Property taxes = $22,000 Property management = $15,000 Maintenance and utilities = $36,000 Swimming pool = $13,000 Professional fees = $8,000 Other expenses = $23,000 What is the property's net operating income? -$74,220 $175,628 $320,460 $348,500

The correct answer is C. Gross rental receipts ($650 × 60 × 12) = $468,000 plus non-rental income ($625 × 12) = $7,500 equals potential gross income (PGI) ($468,000 + $7,500) = $475,500. PGI minus vacancy and collection losses ($475,500 - (.08 × $475,500) ) = $437,460 equals Effective Gross Income (EGI). EGI minus expenses equals net income ($437,460 - $117,000) = $320,460. Net Income + Interest + Depreciation = New Operating Income 320,460 + 0 + 0 = 320,460 Since there is no information regarding interest or depreciation expense, Net income = NOI in this question. Report Content Errors

Your client, Sue, age 35, is covered by a pension plan at work, but her spouse, age 37, is not covered by a pension plan. Her salary is $45,000 and his salary is $50,000. How much will go into his account if he contributes the maximum amount to a maximum funded, matching SIMPLE IRA? $13,500 $19,000 $15,000 $19,500

The correct answer is C. He can contribute $13,500 (2020) and his employer will match $1 for $1 up to 3% of salary ($50,000 × .03 = $1,500). Therefore, maximum contribution is $13,500 + $1,500 = $15,000.

Gloria and her husband, Michael, are each 50% shareholders of K-9 Corporation, a closely-held C corporation. The corporation owns a life insurance policy on Gloria's life, and their son Archibald is the named beneficiary of the policy. K-9 Corporation needs cash flow but wants to minimize taxes. To meet these objectives, the policy should be sold to: A. Archibald. B. A UTMA account for the benefit of Archibald. C. Gloria. D. Michael

The correct answer is C. If a life insurance policy is sold, the transfer-for-value rule generally applies. If the transfer-for-value rule applies to a life insurance policy, the death benefit received from the policy will be subject to income taxes. One exception to the transfer-for-value rule is a transfer of the policy to the insured. Gloria can purchase the policy, thereby meeting the need to raise cash for K-9 Corporation, and since this is an exception to the transfer-for-value rule, there will be no income tax to the beneficiary on the death benefit.

According to ERISA, which of the following is/are required to be distributed annually to defined benefit plan participants or beneficiaries? Individual Benefit Statement. The plan's summary annual report. A detailed descriptive list of investments in the plan's fund. Terminating employee's benefit statement. I, II and IV only. I and II only. II and IV only. III and IV only.

The correct answer is C. Individual Benefit Statements are not required annually for defined benefit plans. They are however, required at least once every three years. Alternatively, defined benefit plans can satisfy this requirement if at least once each year the administrator provides notice of the availability of the pension benefit statement and the ways to obtain such statement. In addition, the plan administrator of a defined benefit plan must furnish a benefit statement to a participant or beneficiary upon written request, limited to one request during any 12-month period. There are no individual accounts in a defined benefit plan, so a specific listing of invested assets is not required.

Which of the following types of funding vehicles is eligible (approved) for TSAs? Fixed Annuity Contracts. Life Insurance policy which develops large cash values. Mutual funds. Variable annuity contracts. Custodial accounts holding individual stocks and bonds. Credit union share account. I, II, III, IV, V and VI. I, III, IV, V and VI. I, III and IV only. I and VI only.

The correct answer is C. Life insurance can only be incidental in the plan. Bank and credit union accounts are not eligible investments. Custodial accounts can only hold mutual funds. TSA can be invested in annuities and mutual funds.

Beth works for MG Inc. and was hired right out of school after graduating with a double major in marketing and advertising four years ago. Beth receives a $12,000 distribution from her designated Roth account in her employer's 401(k) plan as a result of her being disabled. Immediately prior to the distribution, the account consisted of $15,000 of investment in the contract (designated Roth contributions) and $5,000 of income. What are the tax consequences of this distribution? She will have $12,000 of income. She will have $5,000 of income. She will have $3,000 of income. She will have no income tax consequences resulting from the distribution.

The correct answer is C. Non qualified distributions from a designated Roth account associated with a 401k are subject to tax on a pro-rata basis. Her total account is the 15,000 invested and the 5,000 of income for a balance of $20,000. Since 75% (15,000/20,000) of the value in the account consists of basis and the remaining 25% consists of earnings (5,000/20,000), that same ratio of basis to income will apply to the $12,000 distribution. It is not a qualified distribution because she has not held the account for at least five years.

Your client wishes to make a $5,000 contribution to a Roth IRA. The following are sources of income listed on her data form. Which types of income will be used to calculate the maximum allowable contribution? $5,000 from a Limited Partnership interest. $1,500 from a home-based business. $20,000 in municipal bond interest. $3,000 from child support payments. $250 capital gains distributed from a mutual fund. I, II and V II, IV and V II only II and V

The correct answer is C. Only earned income qualifies for contributions into an IRA. All of the other sources of income are forms of unearned income.

The Securities Act of 1933 is best summarized by the following statement: Requires the registration and provides for regulation of investment advisors. The Act regulates securities in the secondary markets. Regulates both initial public offerings and subsequent secondary offerings by a public company. Established the organized securities exchanges.

The correct answer is C. Option "A" - The Investment Advisors Act of 1940 regulates advisors. Option "B" is incorrect because the Act of 1934 regulates the secondary market. Option "C" - The Act of 1933 regulates both IPOs and secondary offerings. Option "D" - The organized exchanges and previously issued securities are governed by the Securities and Exchange Act of 1934.

Quin owns a house in Connecticut and an apartment in New Orleans. Quin spends most of her time in Connecticut, so she sometimes rents out the apartment in New Orleans when she is not there. This year, Quin rented out the apartment for thirty days and personally used the apartment for forty days. How will Quin's rental activity be classified for tax purposes and why? Nontaxable activity because Quin used the apartment personally more than she rented it out. Mixed-use activity because Quin both rented out the apartment and used it personally. Mixed-use activity because Quin rented out the apartment for more than 14 days and personally used the apartment for the greater of 14 days or 10% of the rental days. Rental activity because Quin rented out the apartment for more than 14 days

The correct answer is C. Option "A" is incorrect; this would be a nontaxable activity if Quin had rented the property for less than 15 days. However, Quin rented the property for 30 days, so it is not a nontaxable activity. Option "B" is incorrect because an activity will not be classified as a mixed-use activity just because the owner both rents the property and uses it personally. Option "C" is correct; even though Quin did rent out the apartment for more than 14 days, she also used it personally for the greater of 14 days or 10% of the rental days. Therefore, Quin's rental activity is classified as a mixed-use activity.

Match the following statement with the type of retirement plan which it most completely describes: "A qualified plan which allows employee elective deferrals of 100% of includible salary and has a mandatory employer match" is... A Profit sharing plan. A Money purchase plan. A SIMPLE 401(k). A Defined benefit plan.

The correct answer is C. Profit sharing plans "A" are not contributory. Answers "B" and "D" do not permit employee elective deferrals.

My margin requirements are 50% initial margin and 25% maintenance margin. I purchase a total of 200 shares at $100 per share using full margin amount for the 200 share purchase. Shortly thereafter, share prices fall to $50 per share. What will my margin call be? $1,000 $1,500 $2,500 $5,000

The correct answer is C. Required equity: $50 × .25 = 12.50 per share Actual equity: $50 - $50 = 0 (current price- loan amount) To meet required equity: $12.50 per share × 200 shares = $2,500

The bond investment strategy of "riding the yield curve" involves: Investing equal amounts in short-term and long-term bonds. Investing equal amounts in each of several maturity periods. Investing either short-term or long-term to take advantage of anticipated interest rate changes. Selling bonds with unrealized losses and replacing them with similar bonds.

The correct answer is C. Riding the yield curve refers to the purchase of debt instruments in anticipation of fluctuations in the rates of return on both long and short-term instruments. Rising rates of interest require repositioning a portfolio in advance of the rise in order to avoid significant price drops. These moves are based on anticipated changes in the yield curve.

Which of the following is a correct statement about the income tax implications of employer premium payments for group health insurance? An S Corporation can only deduct 70% of the premiums for all employees. In a sole proprietorship, the premiums for both the owner and the non-owner are fully deductible. If stockholder/employees of a closely held C corporation are covered as employees, the premiums are fully deductible. Premium costs paid by a partnership are passed through to the partner, who can deduct 70% of the costs on their individual tax returns.

The correct answer is C. S Corporations and proprietorships cannot deduct any premiums for group health insurance for owners. Non-owner employee health premiums are fully deductible to both entities. Answer "D" is incorrect because partners are able to deduct 100% of the health insurance premium on their individual tax returns.

Which of the following describe benefits usually available under an employer-provided short-term disability plan? Short-term disability coverage will start on the first day when disability is related to an illness. The definition of disability under short-term disability coverage is defined as the inability to perform the normal duties of one's position. Benefits under a short-term disability plan usually extend for one year. Generally, short-term disability coverage will start after sick pay benefits have been provided to a covered employee. I and II only. III and IV only. II and IV only. I, II and III only.

The correct answer is C. Short-term disability benefits usually start the eighth day of an illness (first day for an accident) and generally last no more than six months.

Complex Corporation is ready to adopt a profit sharing plan for eligible employees. Which of the following groups would have to be considered in meeting the statutory coverage and participation tests? Employees of Simple Corporation, in which Complex owns 85% of the stock. Employees of Universal Corporation, in which Complex owns 55% of the stock. Rank and file workers at Complex who are union members with a contract that provides retirement benefits as a result of good-faith collective bargaining. Employees who are leased and covered by the leasing corporation's profit sharing plan. I only I and II I and IV II and III

The correct answer is C. Simple must be considered because Complex owns more than 80% and the leased employees must be considered because their leasing company's is not a pension plan. Universal would not be considered a subsidiary because it is only 55% not more than 80%. The union employees are excluded from testing by the IRC.

Bob purchased a $400,000 whole life policy on the life of his sister, Lisa. Lisa's son, Steve, is the beneficiary of the policy. Lisa died, and the $400,000 was paid to Steve. Which of the following statements is correct regarding this arrangement? A. The death benefit of $400,000 will be included in Lisa's gross estate. B. Steve will have taxable income of $400,000 because a transfer for value has occurred. C. Bob has given a $400,000 gift to Steve. D. There are no gift or income tax consequences at the time of Lisa's death

The correct answer is C. Since Bob owns the policy and has named Steve as the beneficiary, he will have given a $400,000 gift to Steve upon Lisa's death.

Blake is a CFP® professional and prepares tax returns for his clients. He prepared his brother's income tax return for $1,000 and he willfully neglects to include $30,000 of income since his brother did not receive a 1099 for consulting work. Blake is aware that his brother earned the $30,000 but fails to report it since he doesn't believe the IRS will catch the understatement of income. The additional tax on this $30,000 of income would have been $7,500. How much of a penalty may Blake be subject to for the understatement of income? None, but his brother will be subject to penalties. $3,750 $5,000 $7,500

The correct answer is C. The preparer penalty for willful or reckless conduct is the greater of $5,000 or 50% of the income derived by the preparer for the return. In this case, he charged his brother $1,000.

Which of the following correctly describes the tax implications of a self-funded accident or medical plan where the employer reimburses the employee directly? In a discriminatory plan, the employer cannot deduct the reimbursements paid to the employee. In a discriminatory plan, a highly-compensated employee must include the excess benefit in his or her income. In a non-discriminatory plan, the benefits received by employees are generally tax free without limitation. In a non-discriminatory plan, the employer can deduct reimbursements to the employee if they are paid to the employee or the employee's beneficiary and are considered reasonable compensation. In a discriminatory plan, benefits received by non-highly compensated employees are generally tax free without limit. I, II and IV only. II, III and IV only. III, IV and V only. I, III, IV and V only.

The correct answer is C. Statement "I" is incorrect because the employer can always deduct the premiums. Statement "II" - The highly compensated employees may be required to pay taxes on all or part of the reimbursements.

Which of the following explain the tax ramifications of a non-qualified deferred compensation plan? A participant in an unfunded plan will not be currently taxed if the promise of benefits is unsecured and the agreement is executed prior to the first day of service under the agreement. If a funded plan is established by a general partnership and the benefits for general partners are fully vested, then contributions are deductible to the partnership and each partner receives a pro-rata share of the reduced partnership income. A funded plan with a "Rabbi Trust" will not be currently taxable to the participant, even though vested in the benefits, due to the "substantial risk of forfeiture." Payments to be made to a participant's beneficiary are not included in the decedent's gross estate if the payments are guaranteed. I only. I, II and III only. I and III only. II, III and IV only.

The correct answer is C. Statement "II" is incorrect because any funded plan is taxable unless a Rabbi trust is utilized. Statement "IV" in incorrect because the NPV of the guaranteed future income will be included in the gross estate of the deceased participant.

XYZ company anticipates paying the following dividends, starting next year: Year 1: 2.25 Year 2: 2.75 Year 3: 3.01 After the third year, they anticipate dividends growing at 6%. If Diego's required rate of return is 12%, how much would he be willing to pay for this stock? $35.08 $40.07 $44.20 $47.38

The correct answer is C. Step #1: Apply the constant growth dividend formula to value the stock as of year 3. V = 3.01(1.06) ÷ (.12 - .06) V = 53.18 Step #2: Use uneven cash flows to determine the NPV of the stock at time period zero (today). CF0 = 0 CF1 = 2.25 CF2 = 2.75 CF3 = 3.01 + 53.18 = 56.19 I = 12 NPV = ? Answer: $44.20

A client became age 70 1/2 on October 1, 2019 and must receive a minimum distribution from his IRA account, which had a value (at the end of the prior year) of $48,000. His spouse, age 63, is the beneficiary of the IRA account. The life expectancy according to IRS tables for ages 70 and 71 use the factors of 27.4 and 26.5 respectively. If the client takes a $1,000 distribution by April 1, 2020 what will be the tax penalty, if any, on the first RMD? $0 $83 $376 $416

The correct answer is C. The $48,000 is subject to a 27.4-year life expectancy of the client, thus minimum distribution is $1,752 per year. If the client takes only $1,000, the balance ($752) is subject to a 50% excise tax or $376 penalty.

As a measure for risk, the Capital Market Line (CML) uses the: Risk free rate of return. Beta of the market. Standard deviation of the market. Portfolio weighted beta.

The correct answer is C. The CML (Capital Market Line) uses standard deviation, while the SML (Security Market Line) uses the beta as its "risk" measurement.

Tara is a participant in Kean Co.'s defined benefit plan and standard 401(k) plan. Tara, who is a mid-level manager, is 44 years old and earns $100,000. She has five years of service for purposes of the plans and has worked at Kean for five years. The plan provides a benefit of 2% for each year of service. Both plans have the least generous graduated vesting schedule possible. Almost eighty percent of the accrued benefits in the defined benefit plan are attributable to the rank and file employees, and not the owners. According to the actuary, Tara's accrued benefit in the defined benefit plan is $10,000. Over the last five years, Tara has deferred a total of $30,000 from her salary, which has grown to $40,000. In addition, Kean has matched these contributions with $15,000, which is now worth $20,000. If Tara were to leave today, how much could she rollover into a new employer's plan $70,000 $64,000 $62,000 $57,000

The correct answer is C. The DB plan is not top heavy. Therefore, the vesting for the DB plan is a 7 year graded schedule and he is 60% vested in the $10,000. She is 100% vested in the $40,000, but only 80% vested in the employer matching contribution. This question requires you to know the vesting schedules, which are not provided on the exam. For DB either 5 year cliff or 7 year graded, for DC 3 year cliff or 6 year graded. DBDCYr 10%0%Yr 20%20%Yr 320%40%Yr 440%60%Yr 560%80%Yr 680%100%Yr 7100%-- DB plan has 10,000 and is 60% vested = 6,000 DC Employee contribution is $40,000 (100% vested) DC Employer match is 20,000 and is 80% vested = 16,000 Total vested is $62,000

Johle worked at PKMG consulting firm for the last several years. He started working at PKMG while in college as a paid intern and has now obtained a full time job there. His first year he was 19 and he worked 300 hours. His second year he worked 1500 hours and his third year he worked 2088 hours. Johle is now three quarters of the way through his fourth year. PKMG has three retirement plans with standard eligibility rules and the longest vesting permitted by the IRC. In addition, none of the plans permit in-service withdrawals. The funding of the plans for Johle is as follows: Johle is considering taking another job in Washington and wants to know how much he would forfeit if he were to resign and take another job with another firm today. Money Purchase Pension PlanCash Balance Plan401(k) planPKMG match$500$1,200$300PKMG profit sharing$0$0$600Johle's contribution$0$0$5,000Earnings - PGMG $ (match & profit sharing)$100$600$100Earnings - Johle $$0$0$500PKMG QNEC$0$0$500Total$600$1,800$7,000 $2,720 $2,760 $3,080 $3,480

The correct answer is C. The MPPP uses 2 to 6 graded as does the 401(k) plan. The cash balance plan is required to use 3 year cliff vesting. He has two years of service for purposes of vesting. The first year does not qualify since he did not work more than 1000 hours. Therefore, he can take 20% of the employer funds in the MPPP and the 401(k) plan, but nothing for the cash balance plan. Eligibility and vesting are two different things. Year 1 = 19 Year 2 = 20 - 1st year for vesting Year 3 = 21 - 2nd year for vesting, 1st year eligible Year 4 is ¾ over He has been earning vesting any year he had over 1,000 hours = 2 years. 2 to 6 graded 401(k) Plan: (ER) $1000 × 20% = $200 plus $5,000, plus $500 for earnings and $500 for the QNEC, which is 100% vested by definition. Balance is $7,000 - 6,200 vested = $800 forfeited MPPP - $600 × 20% = $120 Balance $600 - $120 vested = 480 forfeited CB is 3 year cliff. Forfeits all $1,800 The amount he can take is $6,320 (vested) and the amount that he forfeits is $3,080 ($9,400 - $6320). Choice a is wrong because it includes 20% of the cash balance plan. Choice b is wrong because it assumes 3 years of service or 40% vesting - does not include the cash balance amount. Choice d is wrong since it treats the QNEC like the other ER contributions.

1.2% complete This is a Single Choice Question; skip ahead to question content Incorrect Correct answerYour answer A B x C D Confidence Level: Medium Continue Difficulty Level:Difficult Jose has been a generous man his entire life, and has gifted the amount of the lifetime exemption prior to his death. Jose died with the following remaining assets and value: AssetValueValueJanuary 2July 2as of the date of deathas of the alternate valuation datePersonal residence$800,000$600,000Investment amount$1,500,000$1,200,000Annuitized annuity payable to spouse$600,000$550,000Copyright$82,000$75,000Other Assets$150,000$175,000 In addition, Jose's spouse sold the personal residence on May 31 for $700,000 and properly elected the alternate valuation date. What is the value of Jose's gross estate for 706 purposes? $2,600,000. $2,700,000. $2,757,000. $3,132,000.

The correct answer is C. The alternate valuation date is utilized to save on estate taxes. Most assets can utilize the AVD, with the exception of "wasting assets" and assets sold during the estate administration. "wasting assets" are any assets that naturally will decline in value as time passes. Copyrights become less valuable as they come closer to their expiration. Annuitized annuities will decline due to payouts. These "wasting assets" cannot use the AVD, the must use Date of Death value. Any assets sold between DOD and AVD will use the sale price. $700,000 (sale of residence) + $1,200,000 (AVD) + $600,000 (wasting asset - DOD) + $82,000 (wasting asset - DOD) + $175,000 (AVD) = $2,757,000 You got this wrong before because you didn't catch the wasting assets.

A convertible bond your client is interested in buying has a 8% coupon rate, a conversion ratio for 100 shares, while market rates on similar issues are currently 12%. It pays interest twice a year, has a par value of $1,000 with a maturity of 20 years. The current stock price is $8.50 per share. What price should your client expect to pay? $1,125 $1,000 $850 $699

The correct answer is C. The calculated value as a debt instrument here is $699.07, but when the conversion value puts the rate at $850, you can be certain that the bond is selling in the market place at its convertible value of $850, not its debt value of $699. CV = (1,000 ÷ CP) × Ps $850 OR N = 20 × 2 = 40 i = 12 ÷ 2 = 6 PV = ? PMT = (.08 × 1,000) ÷ 2 = 40 FV = 1,000 PV = 699.07 The conversion price is not given; however, (1,000 ÷ CP) is the conversion ratio, which is given (100 shares). Therefore: CV = 100 shares × Ps CV = 100 × 8.50 CV = $850

Which one of the following types of investor benefits most from the tax advantage of preferred stocks? Government. Individual. Corporate. Mutual funds.

The correct answer is C. The corporate dividend-received deductions are based on ownership. TCJA of 2017 updated the amounts. If a corporation owns 20% or less, the have a DRD of 50%. If 20% or more (and less than 80%) of the corporation paying the dividend is owned by the company receiving the dividends, then up to 65% of the dividend is tax free. If ownership is greater than 80% (affiliated corporations) the DRD is 100%.

Barbara Reed owns an LMN, Inc. bond with a par value of $1,000. LMN is a AA-rated bond that matures in 7 years. Barbara receives $55 of interest income from LMN semiannually. Comparable debt, i.e., is AA-rated, 7-year maturity, yields 12%. The bond's duration is 5 years. What is the intrinsic value of the bond? $703.36 $880.80 $953.53 $954.36

The correct answer is C. The intrinsic value of a bond is its calculated present value. N = 7 × 2 i = 12 ÷ 2 PV = ? PMT = 55 ($110 ÷ 2) FV = 1,000

Calculate the maximum contribution for an employee, age 41, earning $140,000 annually, working in a company with the following retirement plans: a 401(k) with no employer match and a money-purchase pension plan with an employer contribution equal to 12% of salary. $19,500 $16,800 $36,300 $57,000

The correct answer is C. The maximum 401(k) plan employee contribution for an individual under age 50, is $19,500 (2020). The 12% money-purchase plan will add $16,800 ($140,000 × .12). Total contribution is $36,300; $19,500 from the employee and $16,800 from the employer.

Carol, age 55, earns $200,000 per year. Her employer, Reviews Are Us, sponsors a qualified profit sharing 401(k) plan, which is not a Safe Harbor Plan, and allocates all plan forfeitures to remaining participants. If in the current year, Reviews Are Us makes a 18% contribution to all employees and allocates $7,000 of forfeitures to Carol's profit sharing plan account, what is the maximum Carol can defer to the 401(k) plan in 2020 if the ADP of the non-highly employees is 1%? $4,000 $19,500 $10,500 $26,000

The correct answer is C. The maximum annual addition to qualified plan accounts is $57,000. If Reviews Are Us contributes $36,000 ($200,000 × 18%) to the profit sharing plan and Lisa receives $7,000 of forfeitures, she may only defer $14,000 ($57,000 - $36,000 - $7,000) before reaching the $57,000 limit. However, she will also be limited by the ADP of the non-highly employees because she is highly compensated (compensation greater than $130,000). If the non-highly employees are deferring 1% then the highly compensated employees can defer 2% (1×2=2). Therefore, she is limited to a deferral of $4,000 (200,000 x 2%). Since she is 50 or older she can also defer the catch up amount of $6,500 which is not subject to the ADP limitation. Therefore, her maximum deferral is $10,500.

XYZ has a noncontributory qualified profit sharing plan with 300 employees in total, 200 who are nonexcludable (50 HC and 150 NHC). The plan covers 75 NHC and 35 HC. The NHC receive an average of 4% benefit and the HC receive 5.8%. Which of the following statements is (are) correct? The XYZ company plan meets the ratio percentage test. The XYZ company plan fails the average benefits test. The plan must and does meet the ADP test. 1 only. 2 only. Both 1 and 2. 1, 2 and 3.

The correct answer is C. The plan does not have to meet the ADP test because it is a noncontributory plan. The ADP test would apply if there were a non safe harbor 401(k) plan. The plan meets the ratio percentage test and fails the average benefits test. Ratio percentage test (75/150) divided by (35/50) equals 71.4% PASSES Average benefits percentage test: 4% / 5.8% = 68.97% FAILS

Judith Nester, CFP®, has begun making investment recommendations for her client, Ollie. Ollie's portfolio is currently comprised of: Position FMV YTD Return QRS Stock (large company) $110,000 9% Large Cap Blend Mutual Fund $120,000 -3% ABC Stock (small company) $200,000 11% Mid Cap Growth Mutual Fund $165,000 4% S&P 500 Index Fund $100,000 6% International Mutual Fund $155,000 -4% Bond Index Mutual Fund $150,000 4% Which benchmark should Judith utilize to measure relative performance of Ollie's current investment portfolio? A. S&P 500. B. MSCI EAFE. C. Wilshire 5000. D. Russell 2000.

The correct answer is C. The portfolio is a blend of small, mid and large company stocks, as well as bonds. The Wilshire 5000 is the best index to use capturing the overall U.S. market. While the client has an international mutual fund, the MSCI EAFE would be inappropriate considering the portion of domestic assets.

Which of the following statements best describes the probation period in a disability income policy? The period of time that must elapse before the policy is activated. The period of time available for the insurer to cancel coverage under the policy. The period of time the insured must wait before specified illnesses or injuries are covered. The period of time the insured must wait before benefits are payable.

The correct answer is C. The probation period, when included in a Disability Income policy, is the time the insured must wait after the issue of the policy before specified conditions will be covered.

The value of the convertible bond as a debt instrument does not depend on which of the following? The bond's coupon. Current interest rates. The conversion price of the bond. The term of the bond.

The correct answer is C. The value of the bond as a debt instrument is considered separately from its convertibility and is calculated using the bond formula or the present value methodology on the calculator. However, should the conversion value be greater than the "debt value" of the bond, it will sell for the higher price.

A client has bought a stock for $40 per share. At the end of the first year, she purchases another share at $43 per share. At the end of the second year with the share price of $48, she sells her shares. Along the way, at the end of each year, she received a $2 per share dividend. What is the time-weighted return on her investment? 9.53% 13.5% 14.3% 16.6%

The correct answer is C. This is simply an uneven cash flow problem. CF0 = <$40> CF1 = $2 CF2 = $50 IRR = 14.33% Note: Since this is a time weighted return, we are only concerned about the security's cash flow. Therefore, we ignore the second purchase at $43 per share.

Jake established a traditional IRA five years ago at a local financial institution. He is considering the purchase of a limited partnership interest with his IRA funds. A CFP® professional would inform him that some of the income earned by the limited partnership may be taxable to Jake because of which rules? A. Front-loading. B. Prohibited IRA investments. C. Unrelated business taxable income. D. At Risk Rule.

The correct answer is C. Unrelated business taxable income is often reported on limited partnership K-1s. This would potentially cause some of the partnership earnings to be taxable currently, even though the partnership interest is held inside an IRA.

Your client's employer has recently adopted a group universal life insurance plan. The advantages of such a plan for your client typically include all of the following EXCEPT that: It allows employees to borrow or withdraw cash. It provides an opportunity to continue coverage after retirement. The entire premium cost is paid by the employer. It provides flexibility in designing coverage to best meet individual needs.

The correct answer is C. Usually, the employee is required to pay part or all of the premium cost of group universal life insurance.

WestN, Inc. sponsors a 401(k) profit sharing plan with a 50% match. In the current year, the company contributed 20% of each employee's compensation to the profit sharing plan in addition to the match to the 401(k) plan. The company also allocated a forfeiture allocation of $4,000. The ADP of the 401(k) plan for the NHC is 4%. Wade, who is age 45, earns $190,000 and owns 19% of the company stock. If Wade wants to maximize the contributions to the plan, how much will he defer into the 401(k) plan? $19,500 $11,400 $10,000 $5,000

The correct answer is C. Wade is highly compensated because he is more than a 5% owner, so the maximum that he can defer to satisfy the ADP Test requirements is 6% (4% + 2%). Wade is also limited by the 415(c) limit of $57,000. Since the company contributes $42,000 (20% of $190,000 + $4,000 of forfeiture allocations), he only has $15,000 to split between the deferral and the match. Thus, he contributes $10,000* and the match is $5,000, which when added to the $42,000 totals $57,000. 6% of his salary of $190,000 is $11,400. However, he cannot defer this amount due to the 415(c) limit. *When he contributes they match 50%, so for every dollar he contributes 1.5 × that amount goes into the plan. Take 15,000/1.5 = $10,000

A rise in the price of the Japanese Yen in relation to the U.S. Dollar results in: A devaluation of the Yen. Excess reserves in the U.S. current account. A revaluation of the Yen. A negative balance of payments.

The correct answer is C. Were the Yen to fall in value against the dollar, this would constitute a devaluation, but when it costs more dollars to buy a Yen, this is considered an appreciation or revaluation of the Yen.

Eric works for Carpets, Inc. Carpets, Inc issued him both ISOs and NQSOs during the current year. Which of the following would be the most compelling reason why they might issue both ISOs and NQSOs? They want to issue over $80,000 in options that are exercisable in the same year. The NQSOs and ISOs are exercisable in different years. The company wants to provide the NQSOs to assist the individual in purchasing the ISOs. Since they are virtually the same there is no compelling reason to issue both in the same year.

The correct answer is C. When both ISOs and NQSOs are available in the same year the individual can exercise and sell the unfavored NQSOs to generate enough cash to purchase and hold the favored ISOs. It would also be valuable to have both if they issued over $100,000 in options exercisable in the same year because there is a $100,000 limit on ISOs.

Wilma is married to Herb, who abandoned her five years ago. She has not seen or communicated with him since June of that year. She maintains a household in which she and her two young dependent children live. Which of the following statements about Wilma's filing status in this year is correct? Wilma can use the rates for single taxpayers for this year. Wilma can file a joint return with Herb for this year. Wilma can file as a head of household for this year. Any of the above.

The correct answer is C. Wilma meets the "abandoned spouse" rules. Therefore, she can file as a head of household. Otherwise, her filing status would be married, filing separately. Head of Household required that the taxpayer pay for more than 50% of the upkeep of the home in which the qualifying individuals reside. The qualifying individuals need not be dependents of the taxpayer.

Which of the following is NOT a factor that affects (and in some cases limits) the liability of the insurer? Insurable interest. Policy face value. Rate structure. Deductibles.

The correct answer is C. Without an insurable interest, the insurance will not be sold. This affects liability of an insurer. Face value is the most (limit) an insurer will pay. Deductibles are paid by the insured, and other insurance is taken into account in all policies but life insurance, where face value limits the amount paid. The rate structure is directly related to establishing premiums, not a liability of the insurance company.

The Federal Reserve is currently tightening the money supply. As the treasurer and CFO of your company, which of the following best describes the hedge position that you should undertake and the reason for taking it to protect your company's long-term bond inventory. A short position to hedge against increases in bond prices. A long position to hedge against increases in bond prices. A short position to hedge against decreases in bond prices. A long position to hedge against decreases in bond prices.

The correct answer is C. You own a long position on the bonds. A tightening of money will cause a rise in the interest rates, thus exposing your bonds to a loss in value when bond prices decrease as a result. You should undertake a short (sell) position in interest rate futures to protect your position.

The market where exchange and broker dealer services are eliminated entirely is: The primary market. The secondary market. The third market. The fourth market.

The correct answer is D. The fourth market is the market where corporation and institutional investors deal directly with one another. Primary market is where investment bankers and corporations meet to arrange offerings to the public. Secondary markets are where previously issued securities are sold (exchanges, etc.). The third market is over the counter.

Which of the following statements accurately describes a QTIP (qualified terminable interest property) trust? A QTIP may have multiple beneficiaries of the income interest as long as the principal beneficiary is the surviving spouse. The QTIP is permitted to accumulate income. The surviving spouse must have a general power of appointment over the trust assets. The trustee may have the power to invade the corpus for the spouse for an ascertainable standard.

The correct answer is D. A QTIP trust qualifies for the unlimited marital deduction. It does not give the spouse an unlimited general power. A QTIP trust must distribute income annually only to the surviving spouse. Only one beneficiary is permitted in a QTIP trust in order to qualify for the marital deduction. The trustee may have the power to invade for the HEMS of the spouse only.

If a bond is trading at its par value, the internal rate of return (IRR) of the bond is equal to the bond's: I. Yield to call. II. Current yield. III. Coupon rate. IV. Yield to maturity. A. IV only. B. I and II. C. III and IV. D. I, II, III and IV.

The correct answer is D. A bond with a price equal to its par value creates a time value of money situation where the present value is equal to the future value. When this is the case, the coupon rate and yield to maturity will be the same. In addition, the yield to call equals the yield to maturity when no call premium is provided.

A non-qualified plan which is subject to the claims of creditors yet is irrevocable and not accessible by the employer is called: A Supplemental Executive Retirement Plan (SERP). A funded deferred compensation plan. An excess benefit plan. A Rabbi trust.

The correct answer is D. A rabbi trust is an irrevocable trust but, unlike a funded deferred compensation plan, the assets are subject to the claims of the employer's creditors. This avoids constructive receipt by the employee and delays income taxation until distribution.

Which of the following are advantages of dividend reinvestment plans? Reinvested dividends currently are tax deferred. They help firms raise new capital. They give investors a systematic way to accumulate capital. Companies build goodwill by offering these plans to shareholders. I and II only. II and IV only. III and IV only. II, III and IV only.

The correct answer is D. All are true with the exception of dividends paid on a mutual fund are taxable, even when reinvested.

Janice, who is single, had gross income of $38,000, and incurred the following expenses: Charitable contributions = $2,500 Taxes and interest on home = $9,000 Legal fees incurred in a tax dispute = $1,000 Medical expenses = $4,000 Penalty on early withdrawal of savings = $200 Her AGI is: $38,000 $21,300 $29,000 $37,800

The correct answer is D. All but penalty for early withdrawal of savings are itemized deductions. The question asks to determine the AGI. Therefore, the AGI is $38,000 less $200 in penalties which equals $37,800.

Eddie Bauer bought a tax-exempt Original Issue Discount (OID) bond in November of 1998. Which of the following statements is/are true? The bond basis increases at a set rate each year. The difference between maturity value and the original issue discount price is known as the OID. The bond's earnings are treated as exempt interest income. The bond was issued at a discount to its par value. II and III only. I and IV only. I, II and IV only. I, II, III and IV.

The correct answer is D. All of the above statements are descriptions of the Original Issue Discount bond.

Based upon the Internal Revenue Code, which of the following statement(s) is/are accurate? Medical expenses paid as a benefit to a surviving spouse are excludable from gross income only to the extent they would have been excluded if they had been paid to the employee. Highly-compensated employees may lose their tax-free status of medical benefits under a self-insured plan which is discriminatory. A highly-compensated employee may be taxed on part of his or her medical expenses for which he or she is reimbursed under a discriminatory self-insured plan, even if the same benefits are available to all workers. I only. II only. I and II only. I, II and III only.

The correct answer is D. All of the statements are accurate.

Services provided on a discounted or free basis to employees are not includible in taxable income to the employee under which of the following circumstances? The employer must incur no substantial cost in providing the service. Services offered to the employees must be in the line of business in which they are working. Services cannot be discounted more than 25% of the price that is available to customers. If there is a reciprocity agreement between two unrelated employers in the same line of business. I and II only. II and III only. III and IV only. I, II and IV only.

The correct answer is D. All statements are correct except for Statement "III". This is because the percentage of discount that is stated is limited to 20%.

Kyle had contributed $20,000 in nondeductible contributions to his traditional IRA over the years. This year the account balance was $52,000 and he made a withdrawal of $5,000. What amount is reported on Kyle's Form 1040? $5,000 only $1,923 only $3,077 only Both $5,000 and $3,077

The correct answer is D. On Form 1040 Kyle will report the total distribution of $5,000 and the taxable amount of the distribution of $3,077 calculated as $32,000 ÷ $52,000 × $5,000. account balance = $52,000 non-deductible contributions = $20,000 (not taxed at distribution) $32,000 would be taxable. Because there is both taxable and non-taxable money, each distribution is a pro-rata distribution of both. 32k/52k = .6154 5,000 × 61.54% = 3,076.92

Your client, a single-filer, has an income of $80,000. Which of the following conditions would prevent a deductible IRA contribution from being made by your client? Participated in a Section 457 deferred compensation plan. No other retirement plans were available to the employee. Made contributions to a 403(b) plan. Received retirement payments from a pension plan at age 65 (no longer an employee at the sponsoring employer). Has account in previous employer's profit-sharing plan. Received no employer contributions. No forfeiture allocations were made. Eligible to participate in a defined benefit plan, but waived participation when it was calculated that employee retirement benefits would be greater with the IRA. I, II and IV only. II, IV and V only. II, III, IV and V only. II and V only.

The correct answer is D. An active participant is an employee who has benefited under one of the following plans through a contribution or an accrued benefit during the year: qualified plan; annuity plan; tax sheltered annuity (403(b) plan); certain government plans (does not included 457 plans); SEPs; or SIMPLEs. Statement I is a non-qualified deferred comp plan (not one of the plans listed above) and therefore not to be taken into consideration for active participation status. Statement II & V are on the list above. For a defined benefit plan, an individual who is eligible for the plan is automatically considered an active participant. Statement "III" is not active participation, rather it is retirement, and Statement "IV" as described without contributions or forfeitures is not "active participation," but a change in conditions regarding employer contributions or forfeitures could stem deductibility of IRA contributions.

All of the following are factors that can influence the amount of employer contribution to an individual's age-based profit sharing plan account EXCEPT: A. Salary. B. Age. C. Discount rate. D. Years of service.

The correct answer is D. An age-based profit-sharing plan skews the employer contributions to favor older employees. The contribution is based on the employee's salary and age, and a discount rate used to determine a present value. The employee's years of service are not considered in the formula. Years of service is used in Defined Benefit Plans.

Sherman, age 52, works as an employee for Cupcakes Etc, a local bakery. Cupcakes sponsors a 401(k) plan. Sherman earns $50,000 and makes a 10% deferral into his 401(k) plan. His employer matches the first 3% deferral at 100% and they also made a 5% profit sharing contribution to his plan. Sherman also owns his own landscaping business and has adopted a solo 401(k) plan. His landscaping business earned $40,000 for the current year. What is the total contribution that can be made to the solo plan, assuming his self-employment taxes are $6,000? $19,500 $21,000 $26,000 $28,400

The correct answer is D. An individual can defer up to $19,500 (2020) plus an additional $6,500 catch up for all of their 401(k) and 403(b) plans combined. Since he is 50 or older he can contribute the 19,500 + 6,500 = $26,000. Since he already contributed $5,000 into his employer plan he can still defer $21,000 ($26,000 - $5,000) into the solo plan. The employer contributions in this question are in addition to the employee deferral limit. Employer contribution into the solo plan: self-employment income $40,000 less 1/2 SE tax $3,000 Net $37,000 X 20% employer contribution $7,400 Total contribution to the solo plan = $21,000 + $7,400

Eldrick, age 40, established a Roth IRA 3 years ago and was tragically struck and killed by an errant golf ball hit by a drunk spectator at a golf tournament. Eldrick had contributed a total of $10,000 to the account and had converted $20,000 from his traditional IRA. His 20 year old son, Charlie, inherited the Roth IRA, which now has a balance of $60,000. Which of the following statements is correct? Charlie can distribute the entire balance from the Roth IRA without it being subject to any income tax or penalty the month after Eldrick dies. Charlie must take minimum distributions from the Roth IRA the year Eldrick dies. If Charlie begins taking minimum distributions, then the first distribution will be partially taxable. Charlie could delay taking a distribution from the Roth IRA for several years and avoid all penalties and income tax on the distribution.

The correct answer is D. Answer a is incorrect because the distribution would not be a qualified distribution since the five year rule has not been met. Answer b is not correct as he could begin taking distribution the year following death or take a full distribution within five years. Answer c is not correct, because the first distribution would consist of previously taxed contributions and would therefore not be taxable. Answer d is correct as he could delay taking a distribution from the account for two years. At that point, the distribution would be qualified since it meets the five-year rule and is on account of death. The distribution would avoid all income tax and penalties.

Jacque's wife just lost her job and they had a death in the family. Jacque is planning on taking a hardship withdrawal from his 401(k) plan to pay for living expenses and funeral costs. Which of the following is correct regarding hardship withdrawals? Hardship withdrawals can be taken even if there is another source of funds that the taxpayer could use to pay for the hardship. Hardship withdrawals are beneficial because although they are taxable, they are not subject to the early withdrawal penalty. Hardship withdrawals can be taken from elective deferral amounts or vested employer contributions. Unless the employer has actual knowledge to the contrary, the employer may rely on the written representation of the employee to satisfy the need of heavy financial need.

The correct answer is D. Answer a is not correct as there must not be another source of funds. Answer b is not correct as they are generally subject to a penalty unless there is an exception under IRC 72(t). Answer c is not correct as a hardship distribution can only be taken from employee deferrals.

A policy which must cover all eligible dependents if the employer pays the entire premium cost best describes: Group term life insurance. Group paid-up life insurance. Group ordinary life insurance. Dependents' group life insurance.

The correct answer is D. Answers "A," "B" and "C" are all benefits for eligible employees, not their dependents. Answer "D" applies to dependents and the employer cannot discriminate.

XYZ company paid a dividend of $3.00 this year and anticipates the dividend to grow each year by: Year 1: 5% Year 2: 7% Year 3: 8% After the third year, they anticipate dividends growing at 6%. If Sydney's required rate of return is 10%, how much would she be willing to pay for this stock? $50.13 $60.57 $70.34 $80.86

The correct answer is D. Step #1: Determine the dividend to be paid each year. Year 1: 3.00 × (1.05) = 3.15 Year 2: 3.15 × (1.07) = 3.37 Year 3: 3.37 × (1.08) = 3.64 Step #2: Apply the constant growth dividend formula to value the stock as of year 3. V = 3.64 (1.06) ÷ (.10 - .06) V = 96.46 Step #3: Use uneven cash flows to determine the NPV of the stock at time period zero (today). CF0 = 0 CF1 = 3.15 CF2 = 3.37 CF3 = 3.64 + 96.46 = 100.10 I = 10 NPV = ? Answer: $80.86

Which of the following is not a requirement for the unlimited marital deduction? In order to qualify for a marital deduction, the decedent must have been married, but could be separated not divorced, as of the date of his death. The surviving spouse must receive property that is included in the gross estate of the decedent. The surviving spouse must be a U.S. citizen unless there is a QDOT. The total value of the qualifying property received by the surviving spouse is excluded from the taxable estate by the unlimited marital deduction.

The correct answer is D. Answers "A," "B" and "C" are all requirements of the unlimited marital deduction. Answer "D" is incorrect because only the net value, not the gross value, of qualifying property left to the surviving spouse is excluded by the marital deduction. The term "net value" for marital deduction purposes equals the gross value of the qualifying property left to the surviving spouse less any taxes, debts, or estate administration expenses payable out of the spousal interest.

Sarah is a 10 percent owner in Canine Connection, LLC, a day-care center for dogs. She is also a 15 percent owner in Little Laughter, LLC, a successful children's clothing store. She does not materially participate in either business. Her at-risk and loss/income for the current year is as follows: Canine Connection-At-risk = $175,000; Loss of $275,000 Little Laughter-At-risk = $25,000; Income of $125,000 She also has wage income of $80,000 and capital gain income of $30,000. Which of the following statements is true? The loss suspended because of the at-risk rules is $75,000 and the loss suspended because of the passive activity loss rules is $75,000. The loss suspended because of the at-risk rules is $75,000 and the loss suspended because of the passive activity loss rules is $0. The loss suspended because of the at-risk rules is $50,000 and the loss suspended because of the passive activity loss rules is $100,000. The loss suspended because of the at-risk rules is $100,000 and the loss suspended because of the passive activity loss rules is $50,000.

The correct answer is D. At RiskPassive LossSuspended Under At RiskPassive Activity Income (Loss)TotalCanine Connection, LLC175275<100><175><275>Little Laughter, LLCIrrelevantIrrelevant0125125 <100K><50K><150K> Loss suspended because of the at risk rules: $275,000 loss - $175,000 at risk = $100,000 suspended. We can net the $125,000 of income against the loss of $275,000, which equals $150,000, minus $100,000 suspended because of the at risk rules, which leaves $50,000 suspended because of the passive activity rules.

If qualified plan eligibility begins after an employee reaches age 21 and completes two years of service, which of the following vesting schedules would be most appropriate? 3-year cliff. 3-7 year graded. 4-40 vesting. Full and immediate.

The correct answer is D. Because eligibility is greater than one year, only Answer "D" is allowed.

What is one characteristic of a Comprehensive Personal Liability (CPL) policy? It generally includes coverage for legal liability that may arise as a result of professional errors and omissions. It provides coverage limited to claims of catastrophic proportions. It provides coverage for legal liability stemming from business activities of the insured by use of a simple extension of coverage amendment. It may be part of a standard ISO homeowners policy or a stand-alone policy.

The correct answer is D. CPL policies are never used to cover Errors and Omissions types of coverage requirements, nor are they used to cover business pursuits.

Which of the following statements concerning cash balance pension plans is correct? The cash balance plan is a defined benefit plan because the annual contribution is defined by the plan as a percentage of employee compensation. The cash balance plan provides a guaranteed annual investment return to participant's account balances that can be fixed or variable and is 100% guaranteed by the Pension Benefit Guarantee Corporation. The cash balance plan uses the same vesting schedules as traditional defined benefit plans. The adoption of a cash balance plan is generally motivated by two factors: selecting a benefit design that employees can more easily understand than a traditional defined benefit plan, and as a plan that has more predictable costs associated with its funding.

The correct answer is D. Cash balance plans are defined benefit plans due to the guaranteed investment returns and benefit formula, not simply a contribution amount. While cash balance plans provide guaranteed rates of return, they are not 100% guaranteed by the PBGC (PBGC has coverage limits). Cash balance plans use 3-year cliff vesting only. Choice d is correct.

Which of the following represents the LEAST favorable means of securing long-term care coverage? Continuing Care Retirement Communities. Disability Income Policy Rider. Association Arrangements. Life Insurance Policy Rider.

The correct answer is D. Continuing Care Retirement communities are structured specifically for Long-Term Care (LTC) coverage, as are the individual policies. Association arrangements are also LTC specific. These three all provide excellent means to obtain LTC coverage. The disability income policy rider takes a coverage that can no longer be carried after age 65 and converts it to useful LTC coverage, another excellent plan. The least favorable is to have diminished coverage that one will most definitely need at some point - life insurance.

For tax purposes, a deduction is allowed for the consumption of the cost of an intangible asset through: Depletion. Depreciation/Cost Recovery. Goodwill. None of the above

The correct answer is D. Cost recovery of an intangible asset is allowed through amortization. Cost recovery and depreciation (one in the same) are applied to tangible assets. The costs of natural resources are recovered through depletion.

Which of the following statements regarding determination letters for qualified plans is true? When a qualified plan is created, the plan sponsor must request a determination letter from the IRS. An employer who adopts a prototype plan must request a determination letter from the IRS. If a qualified plan is amended, the plan sponsor must request a determination letter from the Department of Labor. A qualified plan which receives a favorable determination letter from the IRS may still be disqualified at a later date.

The correct answer is D. Determination letters are issued by the IRS at the request of the plan sponsor. The plan sponsor is not required to request a determination letter. Even if the determination letter is requested and approved, the IRS may still disqualify the plan.

Diana's will leaves all of her property to her husband, George. If he does not survive her by more than 240 days, the property will transfer to Diana's only daughter. Diana dies on May 1 and George dies on the following February 1. Of the following statements, which is true? Diana's property will transfer to her daughter and the property will be eligible for the unlimited marital deduction in Diana's estate. Diana's property will transfer to her daughter and the property will not be eligible for the unlimited marital deduction in Diana's estate. Diana's property will transfer to George and the property will be eligible for the unlimited marital deduction in Diana's estate. Diana's property will transfer to George and the property will not be eligible for the unlimited marital deduction in Diana's estate.

The correct answer is D. Diana's property will transfer to George because he survived her for at least eight months. Therefore, both answer "A" and answer "B" are incorrect. Answer "C" is incorrect because the property that transfers to George will not be eligible for the unlimited marital deduction in Diana's estate because the survivorship clause exceeds 6 months.

Which of the following statement(s) concerning Unrelated Business Taxable Income (UBTI) is/are accurate? Dividends, interest, and other types of income derived from investments in a business are not subject to UBTI. A partnership interest in an investment enterprise, whether active or passive, is subject to UBTI. A direct business activity carried on for the production of income is considered a trade or business for UBTI purposes. Securities of the employer purchased with loan proceeds by an Employee Stock Ownership Plan (ESOP) are not subject to UBTI. I only. I, II and III only. II, III and IV only. I, III and IV only.

The correct answer is D. Direct investment in a business generates income which is UBTI. Any investment which is purchased with "leverage" or borrowed funds generate UBTI except for a qualifying ESOP or LESOP.

When would you advise a person not to wait to exercise a nonqualified stock option? When long-term price appreciation is anticipated, but uncertainty regarding future stock price remains. When the individual has had an excellent year resulting in much higher than expected income. When the price of the stock in the market is out-of-the-money and not expected to enter or change any time soon. When the stock price seems to have peaked and sale will immediately follow exercise.

The correct answer is D. If all gain has been apparently made in a security, rather than lose the profit, and since there are no special advantages to holding non-qualifieds, it may be the time to exercise and to follow with an immediate sale. The rest of the options are actually reasons for holding the option without exercising it.

According to fundamental analysis, which phrase best defines the intrinsic value of a share of common stock? The par value of the common stock. The book value of the common stock. The liquidating value of the firm on a per share basis. The discounted value of all future dividends

The correct answer is D. Intrinsic value is the discounted value of a future stream of cash flows. In the case of a stock, its dividends.

Which of the following would be considered an "incidental benefit" if offered through a defined contribution plan? Term life insurance coverage with premiums equal to 30% of the total cost of benefits. Universal life insurance coverage with premiums equal to 34% of the total cost of plan benefits. Ordinary life insurance coverage with premiums equal to 57% of the total cost of plan benefits. Term life insurance coverage with premiums equal to 25% of the total contributions to the participants account.

The correct answer is D. Life insurance in a qualified plan is limited, under the incidental benefit rule, to 25% of aggregate contributions to the participant's account for Term and Universal life plans. Whole life plans may constitute 50% of the contribution.

The theory of the Yield Curve that attempts to explain the yield curve based upon future rates of inflation is the: Liquidity Preference Theory. Market Segmentation Theory. Short-Wave Theory Expectations Theory.

The correct answer is D. Liquidity preference states that investors prefer liquidity, therefore, short-term money pays less. The market segmentation theory states that supply and demand explain at various maturities the shape of the yield curve

Joseph wants to take a loan from his 403(b) plan. Which of the following requirements will apply to his situation? The loan can exceed $50,000 only if the account is worth more than $100,000. The loan must be paid back with quarterly payments over three years, unless used to purchase a principal residence. The loan is exempt from interest charges since the participant is borrowing his own money. The plan document may provide for a maximum and minimum loan amount.

The correct answer is D. Maximum loan amount is lesser of 50% of vested amount or $50,000 paid in quarterly (or more frequent) payments over five years, unless used for home purchase. Loan must carry reasonable interest rate.

Alisha Marie has just purchased a bond with a $1,000 par value that is callable in 7 years at $1,200. The bond matures in 20 years and has an 11.5% coupon, payable semi-annually. She purchased the bond at a discount at a price of $900. What is the yield to call on this bond? 7.79% 9.91% 13.77% 15.57%

The correct answer is D. N = 7 × 2 = 14 i = ? PV = <900> PMT = .115 × 1,000 × .50 = 57.50 FV = 1,200 i = 7.7866 × 2 = 15.57

Bob and Betty have approached you looking for the right hedge against possible, expected future inflation. You suggest to them that they: Invest in technology stocks. Invest in commodity futures. Invest in long-term U.S. Treasury issues. Invest in precious metals.

The correct answer is D. None of the choices are necessarily stellar, but in contrast to the other choices, Option "D" makes far more sense, as metals have generally performed well as inflation hedges over time.

Todd is an aggressive investor who invests exclusively in stocks. He studies odd-lot theory and head-and-shoulder charting patterns when selecting appropriate stocks for investment. He also knows several members of the board of directors of a publicly-traded company and believes inside information he obtains from them will help with his purchase and sell decisions. Todd's use of odd-lot theory and head-and-shoulder charting patterns is consistent with: A. The semi-strong form of the efficient market hypothesis. B. The weak and semi-strong forms of the efficient market hypothesis. C. The strong form of the efficient market hypothesis. D. None of the forms of the efficient market hypothesis.

The correct answer is D. Odd-lot theory and the January effect are both examples of technical analysis. No form of the efficient market hypothesis supports technical analysis.

Which one of the following incidental benefit rules apply to life insurance coverage provided by a profit sharing plan? Permanent life insurance, accident insurance, or severance benefits may be included as part of the coverage. The 25% incidental benefit cost rule is based on the portion of the premium allocated to the policy's cash value. An employee's costs associated with the purchase of life insurance represent a non-deductible expense. The cost of whole life insurance must be not more than 50% of the total employer contribution allocated to a participant's account.

The correct answer is D. Only life insurance may be included in a qualified retirement plan - no accident, severance, or health benefits may be offered under the incidental benefit rules. To qualify under the incidental benefit rules, the entire premium for universal life cannot exceed 25% of the employer's aggregate contributions, and 50% for whole life insurance. Any pension contributions used to purchase life insurance inside a qualified plan are deductible to the employer.

Which of the following investment strategies involves both very long-term bonds and very short-term bonds for a portfolio, and very few intermediate-term bonds? Bond ladder strategy. Immunizing the portfolio. Bond swap strategy. Bond barbell strategy.

The correct answer is D. Option "A" - Bond laddering consists of periodic maturities including short term, intermediate and long term bonds. Option "B" - Immunizing the portfolio uses duration rather than maturity as a measure of position for implementing the strategy. Option "C" - Bond-swap strategies trade different and varied maturities to meet the objective of the portfolio.

Prairie Dog Corporation (PDC), an oil drilling company, has a "key-person" variable universal life policy on Digger Phelps, its vice-president of drilling operations. The owner and beneficiary of the policy are the corporation. Which of the following is correct? Premiums paid by PDC are taxable income to Digger. Premiums paid by PDC are considered gifts to Digger. Premiums paid by PDC are tax deductible as a business expenses. Any death benefit paid will be nontaxable to PDC.

The correct answer is D. PDC is the owner and beneficiary of the policy. For the same reason, premiums are NOT considered a gift or taxable to Digger, nor will they appear in his gross estate. "Key person" life premiums are not deductible as a business expense. Any death benefit paid will be nontaxable to PDC.

Timothy is covered under his employer's Defined Benefit Pension Plan. He earns $500,000 per year. The Defined Benefit Plan uses a funding formula of Years of Service × Average of Three Highest Years of Compensation × 2%. He has been with the employer for 25 years. What is the maximum contribution that can be made to the plan on his behalf? $137,700 $230,000 $285,000 It is indeterminable from the information given.

The correct answer is D. Read the question carefully. The question asks "what is the maximum contribution that can be made." Remember that for a defined pension plan the contribution must be whatever the actuary determines needs to be made to the plan.

Lien, age 35, recently left his employer, GoGoRoller, a roller blade manufacturer. He left after 18 months because the working conditions were unbearable. GoGoRoller sponsored a SIMPLE IRA. Lien deferred $3,000 into the plan during his time there and the employer contributed $1,500. When he terminated he withdrew the entire account balance of $4,750. Assuming he is in the 12% tax bracket, what is the tax and penalty consequence for this distribution? $570.00 $775.00 $1,187.50 $1,757.50

The correct answer is D. SIMPLE IRAs require a 25% penalty for early withdraws in the first two years if the participant does not meet any of the early withdrawal exceptions. He does not meet any of the exceptions and the distribution is within the first two years. The breakdown of employee deferrals, employer contributions and earnings is irrelevant. Contributions on behalf of Lien were $4,500 plus $250 of growth for a total of $4,750 in the account. Therefore, his tax and penalty consequence is $1,757.50 = $4,750 × 37%. The 37% is represented by 12% tax plus 25% penalty.

Vijai, age 40, recently left his employer, GoGoRoller, a roller blade manufacturer. He left after 10 years because the working conditions became unbearable. GoGoRoller sponsored a SIMPLE IRA. Vijai deferred $30,000 into the plan during his time there and the employer contributed $15,000. When he terminated he requested the entire account balance of $55,000. How much would his check have been for? $41,250 $44,000 $45,000 $55,000

The correct answer is D. Simple IRAs do not require the 20% withholding because they are not qualified plans. Therefore, the entire account balance would have been distributed to him. The early withdrawal penalty is not assessed at the time of distribution from the qualified plan, but on the 1040.

Luna Autobody wants to establish a pension plan. They want the employee's to bear the investment risk and favor older employees. Which plan should they establish? Age based profit sharing plan. Cash Balance Pension Plan Money Purchase Pension Plan Target Benefit Pension Plan

The correct answer is D. Since they want to establish a pension plan where the employee's bear the investment risk and favors older employees they should establish a target benefit plan. An age based profit sharing plan is not a pension plan - it is a profit sharing plan. The employer bears the risk on the cash balance plan. The money purchase plan favors younger employees.

Which of the following are characteristics of a non-qualified deferred compensation agreement for an individual? It may provide for benefits in excess of qualified plan limits. The contribution underlying the agreement may NOT be structured as additional compensation to the employee. It must be entered into prior to the rendering of services to achieve deferral of compensation. The contribution underlying the agreement may be paid from the current compensation of the employee. I and II only. I and III only. II and III only. I, III and IV only.

The correct answer is D. Statement "II" is incorrect. The underlying contribution may be structured as additional compensation (a so-called salary continuation plan.) Statement "IV" represents a so-called pure deferred compensation plan.

Which of the following are correct statements about self-employed retirement plans? I. Benefits provided by a self-employed defined benefit plan cannot exceed the lesser of $230,000 or 100% of income in 2020. II. May be established by an unincorporated business entity. III. Contributions to "owner-employees" are based upon their gross salary. IV. Such plans are permitted to make loans to common law employee participants. I and II only. I and III only. II and IV only. I, II and IV only.

The correct answer is D. Statements "I", "II" and "IV" are correct. Loans are available to the common law employees of the firm. Statement "III" is incorrect because owner-employee contributions are based upon total earned income in the business, not just "salary." (Note: Remember S corporation owners are considered common law employees, so their contribution is based solely on salary and cannot include amounts for dividends or pass-through earnings shown on Schedule E of the 1040 form.)

Farmer Fred wants to retire in 20 years when he turns 64. Fred wants to have enough money to replace 75% of his current income less what he expects to receive from Social Security at the beginning of each year. Fred's full benefit at age 67 is $25,000 in today's dollars. Fred is conservative and wants to assume a 7% annual investment rate of return and assumes that inflation will be 3% per year. Based on his family history, Fred expects that he will live 30 years in retirement. Fred just received his brokerage account statement, which he is using to fund his retirement, and it has a balance of $340,596.44. If Fred currently earns $100,000 per year, approximately how much does he need save at the end of each of the next 20 years to fund his retirement? $6,486 $10,350 $11,215 $12,000

The correct answer is D. Step 1: Determine amount to be funded $100,000 income today 75% WRR $75,000 needs ($20,000*) less social security & pension. $55,000 amount to be funded Step 2: inflate funds to retirement age PV ($55,000), N 20, i 3.00%, Pmt 0, = FV $99,336.12 Step 3: PV of retirement annuity Pmt $99,336.12, N 30, i 3.8835%, FV 0, = PV ($1,809,946.67) Step 4: Annual funding amount FV $1,809,946.67, N 20, i 7.00%, PV ($340,596.44), = Pmt ($12,000) *For this problem you must account for the fact that he plans to retire at age 64, his benefit is stated at age 67. Thus, you will have a reduction in social security retirement benefits: Reduced by 5/9 for each month, for the first three years that a worker retires early. Reduced by 5/12 for each month beyond three years. In this case you are starting three years early so you will have a 20% reduction (5/9 × 36). 20% × $25,000=$5,000 $25,0000-$5,000=$20,000 Uneven Cash Flow Method: $100,000 × .75 WRR = 75,000 <20,000> SS = 55,000 Step #1 NPV at 44 0 CFj} Should equal 00 CFj19 Nj55,000 CFj30 Nj1.07 ÷ 1.03 = -1 = × 100 =i/YRNPV Step #1 Answer = 467,724.6128 Step #2 Savings Required N = 20 i = 7 PV = 467,724.61 - 340,596.44 PMT = ? FV = 0 Answer = 11,999.99

Haley Mills has been a client of yours for quite some time. She recently unearthed some share certificates that her grandmother had left to her a few years ago. These shares that Grandma Mills bought were from an Internet start up. The timing was perfect, because the firm was about to undertake another stock offering and Haley had preemptive rights. Of the firm's initial 1,700,000 share offering, Grandma Mills had invested enough to buy 10,000 shares at $11 per share. The new offering was an 850,000 share offering at $87 per share. If Haley fully exercised her preemptive rights, how much total cash would she pay for the shares in this new offering? $55,000 $217,500 $385,750 $435,000

The correct answer is D. The 10,000 share purchase of the 1,700,000 share initial offering was .59% (10,000 ÷ 1,700,000). That amount relative to this new offering of 850,000 shares was equivalent to 5,000 (850,0000 × .59%) shares if all preemptive rights were exercised. This 5,000 shares times the $87 offering price means that to fully exercise this right, Haley would require $435,000.

Which of the following are disadvantages or "costs" of the 2503(c) trust? The 2503(c) trust is irrevocable and the grantor must relinquish control. The 2503(c) trust can have only one beneficiary, meaning that funds cannot be taken away from a child who is not observing the wishes of the grantor. The 2503(c) trust requires mandatory distribution of income on an annual basis. The 2503(c) trust has expenses involved in filing tax returns and estimated quarterly tax payments. I and II only. I, III and IV only. II, III and IV only. I, II and IV only

The correct answer is D. The 2503(c) trust does not require annual distribution of income. It is the 2503(b) trust that has this requirement.

John was married to Holly. All of their jointly held assets were community property. Recently Holly died. John was Holly's only legatee. They had 2 children, Patrick (deceased) and Mary, age 44. John wrote a will some years ago and included was a testamentary trust. Which of the following assets of Johns will be included in the testamentary trust as of John's death? An IRA with Holly named as the beneficiary and Mary the contingent beneficiary. One half of a Qualified Plan that named Holly as the beneficiary with no contingent beneficiary. All of the assets included in an intervivos trust created by Holly for her seperate property with John as the only income beneficiary and the children living as remainder beneficiaries. None of the above choices.

The correct answer is D. The IRA will pass via contract law. All not half of the Qualified Plan will be included in the testamentary trust - no named beneficiary. None of the intervivos trust will be included in probate.

Your client has asked you to assist her in examining possible additions to her bond portfolio. She has expressed a desire to minimize risk at this stage in her planning process, and to assure income beginning at the point of her retirement, and lasting throughout. She has a tentative retirement date in seven years at age 65. She will then have an eighteen year life expectancy. Which of the following is an appropriate addition to her current portfolio? 25-year AAA-rated corporate bonds with a seven-year maturity. 20 year AAA-rated municipal bonds with a seven-year duration. 25-year AAA-rated corporate zeroes with a seven-year duration. 20-year US Treasury zeroes with a seven-year maturity. 25-year AAA-rated corporate bonds with a seven-year duration. I, III and V only. II, III and V only. III and IV only. V only.

The correct answer is D. The client is looking for income to begin in 7 years. Therefore anything maturing in 7 years will not provide that income. Zeroes provide no income. She wants something out 25 years, not 20 years. Thus, option "V" is the only appropriate answer.

XYZ Corporation is a closely held corporation. Martin McFly, along with the three other owners, set up a stock redemption agreement requiring the corporation to buy all shares of a deceased or disabled shareholder. The plan is funded by entity life insurance policies on each shareholder. Premiums are paid by the corporation. The agreement states that the share price will be established by an independent, competent third party appraiser. What are the tax implications of this plan? A deceased shareholder's gross estate will be increased by the amount of the life insurance. There is no step-up in basis for decedent's family on the shares of stock covered by the plan. The corporation will owe income tax on the difference between the cash value of the policy and the death benefit amount. I, II and III only. I and III only. II only. None of the above

The correct answer is D. The deceased shareholder's estate will not increase due to the life insurance, as the deceased shareholder does not own the policy and already has the value of his interest in his gross estate. There is a step-up in basis because the decendant died and the shares are "purchased" by the corporation. The corporation is "owed" the premiums by the individual at death and does not pay tax.

Which of the following tasks are the primary responsibilities of a plan trustee? Determining which employees are eligible for participation in the plan, vesting schedule, and plan benefits. Preparing, distributing, and filing reports and records as required by ERISA. Investing the plan assets in a "prudent" manner. Monitoring and reviewing the performance of plan assets. I and III only. I and II only. II and IV only. III and IV only.

The correct answer is D. The duties explained in Statements "I" and "II" are responsibilities of the plan administrator.

Holly bought a stock at the minimum margin, when the stock was trading at $10. The stock paid quarterly dividends of $.25. Holly held the stock for one year and sold the stock when it was trading at $11. What was Holly's holding period return? 10%. 20%. 30% 40%.

The correct answer is D. The first key to this question is knowing that the "minimum margin" is 50%, which is established by the Federal Reserve. So, Holly is required to pay $10 × .50 = $5 in cash and borrow the other $5 per share to make the investment. The question does not reference any margin interest, so it's excluded from the calculation. The second key to this problem is that the Purchase Price in the numerator reflects both the equity contribution of $5 per share and the $5 per share that must be repaid to the broker. The Purchase Price in the denominator only needs to reflect the $5 in equity paid. HPR = (SP - PP +/- CF) ÷ PP HPR = ($11 - $10 + ($.25 × 4) ÷ ($10 × .5) HPR = 40%

Inland marine insurance covers all of the following except: Imports. Floaters. Domestics shipments. The hull of the ship.

The correct answer is D. The hull of the ship is covered under ocean marine insurance. An inland marine insurance is a category of insurance that protects against property losses to goods in transit.

Gia, age 45, is married and has two children. Her employer, Print, Inc sponsors a target benefit plan in which she is currently covered under. Which of the following statements is true regarding her plan? She can name anyone she wishes as her beneficiary. A target benefit plan favors younger employees. A target benefit plan is covered under PBGC. The investment risk is on the employee.

The correct answer is D. The investment risk is on the employee because this is a defined contribution plan. She can only name someone other than her spouse if she has a valid waiver signed by the spouse. This applies to all pension plans. A target benefit plan favors older entrants. A target benefit is not covered under PBGC.

A client must receive a minimum distribution from his IRA account. The value of the account at the beginning of the current year was $53,000. His spouse, age 63, is the beneficiary of the IRA account. The applicable divisor for his distribution is 26.5. If the client takes a $1,000 distribution, what is the tax penalty, if any? $0 $100 $150 $500

The correct answer is D. The minimum distribution is calculated by taking the account value at the beginning of the year ($53,000 in this case) and dividing by the applicable divisor (life expectancy 26.5 years in this example). The resulting figure is the minimum distribution ($2,000). Because the amount distributed was $1,000 less than required under minimum distribution rules, a 50% penalty applies. 50% of the undistributed minimum distribution is $500.

Your client, a 35% owner of a regular C corporation, wants to take out a loan from the company sponsored profit-sharing plan. In order for the loan not be a prohibited transaction, which of the following conditions must apply: Loans are available to all participant/beneficiaries on a reasonably equivalent basis. Have a reasonable rate of interest. Made in accordance with specific plan provisions. Must be adequately secured. II only. I, II and IV only. None of the above. All of the above.

The correct answer is D. This is an owner of a regular corporation (C Corporation.) Owners of C Corporations are eligible for loans as long as the safe-harbor rules are maintained. The items listed in Statements "I," "II," "III," and "IV" are the safe-harbor rules.

Your client is considering the two stocks described below. Assume for this question that the risk-free rate is 6%, the expected return on the market is 14%, and the market's standard deviation is 18%. Stock A price per share = $18 Stock A annual dividend = $2 Stock A dividend growth rate = 3% Stock A Beta = 1.1 Stock A standard deviation = 21% Stock A realized return over past 12 months = 15% Stock B price per share = $12 Stock B annual dividend = $1.50 Stock B dividend growth rate = 4% Stock B beta = 0.88 Stock B standard deviation = 14% Stock B realized return over past 12 months = 12.5% Which stock would you recommend your client purchase, and why? Stock A, because its intrinsic value is greater than Stock B's. Stock A, because its required return is greater than Stock B's. Stock A, because its risk-adjusted return is greater than Stock B's. Stock B, because it is selling for less than its intrinsic value.

The correct answer is D. Though there is a great deal of information here, one should be looking at the intrinsic values of the stocks and comparing them to the market prices of the stocks for over valuation or under valuation. A market selling price above intrinsic value is overvalued. A market price below the intrinsic value means the stock is undervalued and may be considered for purchase. The first step in this problem is to compute the required rate of return using the CAPM formula: R = Rf + B * (Rm - Rf) Stock A =14.8% = 6 + 1.1 * (14-6) Stock B = 13.04% = 6 + .88 * (14-6) Now that we have the required rate of return on the securities based on their volatility, I would utilize the Gordon Growth Model (or dividend model) to determine the price that the security should be at to be "fairly valued." V = D * (1 + g) / (R - g) Stock A =17.45 = [2 * (1+.03)] / (0.148-.03) Stock B = 17.25 = [1.5 * (1+.04)] / (0.1304-.04) So the "fair value" for our investor is $17.45 for stock A and it is trading at $18. Therefore the stock is overvalued (trading higher than its value). Stock B is valued at $17.25 for our investor and is trading at $12. Therefore the stock is undervalued (trading lower than its value).

Using the constant growth dividend valuation model, calculate the intrinsic value of a stock that pays a dividend this year of $2.00 and is expected to grow at 6%. The beta for this stock is 1.5, the risk free rate of return is 3% and the market return is 12%. $48.27 $35.33 $28.75 $20.19

The correct answer is D. Use the constant growth dividend model to solve for intrinsic value. The question does not provide the required rate of return, however the capital asset pricing model can be used solve for required rate of return. V = D1/(r - g) V = 2 (1.06) / (.165 - .06) V = $20.19 R = Rf + b(Rm - Rf) R = .03 + 1.5(.12 - .03) R = .165

Anson Crowe has a 20% required rate of return. He is considering investing in XYZ, Inc., which pays an annual dividend of $.64 and is projected to increase its earnings and dividends by 17% annually. The current market price is $36.50. What is the expected rate of return from XYZ stock and is it undervalued or overvalued given Anson's requirements? 15.0%; overvalued. 17.5%; undervalued. 18.2%; overvalued. 19.05%; overvalued.

The correct answer is D. Use the expected rate of return and the intrinsic value formulas to calculate the correct answer to this problem. Expected rate of return: Er = (D1 ÷ P) + g Intrinsic Value: V = (D1) ÷ (r - g) D1 = .64 × 1.17 = .7488 Expected Return = (.7488 ÷ 36.50) + .17 = .1905 or 19.05% Intrinsic Value = .7488 ÷ (.20 - .17) = $24.96

Your client owns a DGL Corporation convertible bond that has a coupon rate of 8% paid semiannually and matures in five years. Comparable debt yields 7% currently. The GGL bond is convertible into 22 shares of common stock. The current market price of the underlying stock is $52. What is the conversion value of this convertible bond? $925 $1,000 $1,042 $1,144

The correct answer is D. Use the formula for calculating conversion value of a bond. Keep in mind the conversion value may be different from the intrinsic value of the bond (which in equilibrium is the market price of the bond). CV = (PAR ÷ Cp) x Ps CV = (1,000 ÷ 45.45) x PsCV = $22 x $52 CV = 1,144​Cp = conversion price: PAR / shares (1,000 / 22 = 45.45)Ps = Price of the stock

Jim and Anne Taylor are baby boomers who would like to add an equity investment to their portfolio. They require a 12% rate of return and are considering the purchase of one of the following two common stocks: Stock 1: Dividends currently are $1.50 annually and are expected to increase 8% annually; market price = $35Stock 2: Dividends currently are $2.25 annually and are expected to increase 7% annually; market price = $50 Using the dividend growth model, determine which stock would be more appropriate for the Taylors' to purchase at this time: Stock 2, because the stock is undervalued. Stock 2, because the return on investment is greater than the Taylor required rate of return. Stock 1, because its dividend growth rate is greater than Stock 2's growth rate. Stock 1, because the expected return on investment is greater than the Taylor required rate of return.

The correct answer is D. Use the intrinsic value formula to determine whether the stock is over valued or under valued. Then use the expected rate of return formula to determine whether the stock meets the investor's required rate of return. r = D0(1 + g) + g (P) Stock 1 = $1.50 (1.08) + .08 =12.63% $35 Stock 2 = $2.25 (1.07) + .07 =11.82% $50

If an employer sponsors a cash balance plan which guarantees an investment return that is lower than the actual investment results, what will be the effect to the employer? A. The employer will be required to make a non-elective contribution to the plan. B. The employer's required contribution to the plan will not be affected. C. The employer's required contribution to the plan will be increased. D. The employer's required contribution to the plan will be lowered.

The correct answer is D. cash balance plan, the employer will have a lower contribution requirement when actual investment results exceed the guaranteed rate of return in the plan.

Which of the following are considered disqualified persons in regard to retirement plans? fiduciary of the plan person servicing the plan an employer an employee organization a family memeber all of the above

all of the above they are disqualified from engaging in the prohibited transactions

What is another term for a stock redemption?

an entity buy-sell agreement the corporation agrees to buy the deceased partner's ownership interest

Which of the following statements concerning tax considerations of nonqualified retirement plans is/are correct? Under IRS regulations an amount becomes currently taxable to an executive even before it is actually received if it has been "constructively received." Distributions from nonqualified retirement plans are generally subject to payroll taxes. I only. II only. I and II. Neither I or II.

he correct answer is A. Statement "II" is incorrect because payroll taxes are due on deferred compensation at the time the compensation is earned and deferred, not at the date of distribution. Statement "I" is a correct statement.

Which plans listed below are subject to the Qualified Joint and Survivor Annuity requirement? Cash balance plan. Target benefit plan. Defined benefit plan. Money purchase plan. I only. I, II and III only. II, III and IV only. I, II, III and IV.

he correct answer is D. Pension plans are required to offer a QJSA to participants. All of these plans are pension plans. A profit sharing plan is not subject to QJSA requirements.

List the plans in which participation deems the participant to be "active."

qualified plan; annuity plan; tax sheltered annuity (403(b) plan); certain government plans (does not included 457 plans); SEPs; or SIMPLEs.


Ensembles d'études connexes

english study guide chapters 4-6

View Set

Anterior Cruciate Ligament Sprain

View Set

science prokaryotes and eukaryotes

View Set

Physical Science Solar System Quiz

View Set